Sei sulla pagina 1di 118

Lower Secondary

Lower Secondary
Express/Normal( A )
2nd Edition

The Science Matters for Express/Normal( A ) Practical Book


( 2nd Edition ) is designed to complement the Science Matters for
Express/Normal(A) (2nd Edition) textbook. It is written in line with the 2013
Lower Secondary Science syllabus from the Ministry of Education, Singapore. 2nd Edition

Practical Book Teacher's Edition


The practical book enhances the understanding of concepts that students learnt
and provides opportunities for the development of skills.

In the practical book, hands-on activities are used to encourage students to


explore scientific concepts through experimentation. These hands-on activities
develop process skills that students need in order to carry out scientific inquiry.
Selected experiments begin with real-life contexts so that students can develop
an understanding of how the abstract scientific concepts learnt are connected to
real-life occurrences.

Develops process skills in students


The skills developed in each activity are clearly outlined so that students are
made aware of the process of scientific inquiry that occurs in the activity.

Volume B
This leads to a greater understanding of scientific knowledge and promotes the
spirit of inquiry in students.

Promotes Science as an inquiry

2nd Edition
The activities encourage students to discover concepts on their own through
experimentation. These activities have been carefully designed to develop
students to think in a systematic and logical manner about the world
around them.

J Fong • Lam P K • E Lam • C Lee • Loo P L


Other components
• Textbook
• Workbook

Joan Fong • Lam Peng Kwan • Eric Lam


ISBN 978-981-01-1747-4 Christine Lee • Loo Poh Lim

Practical Book
Volume B Teacher's Edition
(S)LSSBPBTE_Cover.indd 1 9/12/12 5:30 PM
Lower Secondary

2nd Edition

Practical Book
Volume B
Teacher's Edition

Joan Fong • Lam Peng Kwan • Eric Lam


Christine Lee • Loo Poh Lim

(S)LSSBPBTE_TitlePage.indd 1 8/24/12 7:10 PM


© 2008, 2013 Marshall Cavendish International (Singapore) Private Limited

Published by Marshall Cavendish Education


An imprint of Marshall Cavendish International (Singapore) Private Limited
Times Centre, 1 New Industrial Road, Singapore 536196
Customer Service Hotline: (65) 6411 0820
E-mail: tmesales@sg.marshallcavendish.com
Website: www.marshallcavendish.com/education

First published 2008


Second edition 2013

All rights reserved.

No part of this publication may be reproduced, stored in a retrieval system


or transmitted, in any form or by any means, electronic, mechanical,
photocopying, recording or otherwise, without the prior permission
of the copyright owner. Any requests for permission should be
addressed to the Publisher.

Marshall Cavendish is a trademark of Times Publishing Limited.

ISBN 978-981-01-1747-4

Printed in Singapore by Times Printers, www.timesprinters.com

Acknowledgements
Book B
Cover Chapter 16
idea tree in field © Rolffimages | Dreamstime.com, 57 dog whistle © Jf530 | Dreamstime.com
people © Marshall Cavendish International (S) Pte Ltd
Chapter 17
Chapter 13 61 telescope © Moreno Soppelsa | Dreamstime.
15 USB connectors © Wikimedia Commons | Public com
Domain, 19 multimeter © Versh | Dreamstime.com,
25 sound mixer © Laputin | Dreamstime.com Chapter 18
77 dyeing hair © Olena Sokalska | Dreamstime.
Chapter 14 com
29 motorcycle damper © David Freund | iStockphoto.
com, 37 Maglev train © Irfan Nurdiansyah | Chapter 19
Dreamstime.com, 41 suction lifter © stockthor | 105 girl jogging in a park © Martinmark |
iStockphoto.com Dreamstime.com

Chapter 15
47 flywheel energy storage © Wikimedia Commons
| Public Domain, 51 electric torchlight © Bigrock |
Dreamstime.com

(S)LSSBPBTE_TitlePage.indd 2 8/24/12 7:10 PM


Preface
The Practical Book is an integral part of the exciting Science Matters (2nd Edition)
Express/Normal (Academic) package that covers the latest Ministry of Education syllabus.
The Practical Book is designed to provide an experiential, hands-on approach to the learning of
Science through carefully selected activities. The activities help students develop process skills,
including interpreting experimental data, and making logical conclusions. Additional questions
are also included to help students reinforce and consolidate the concepts learnt.

Activities are introduced with real-life Clearly drawn diagrams and step-by-step
contexts, where appropriate, to connect instructions facilitate the execution of
the learning and understanding of key the activities.
concepts to the real world. Chapter 14

Name: ( )
Part B: Action of Amylase on Starch
Class: Date:
1. Add 3 cm3 of amylase solution into a test tube. Boil the solution gently for three minutes and
Activity
14.1 Measuring Forces cool it under a tap. Label this test tube ‘C’.
water
Application of Forces
Interactions Through the

Aim
• To measure forces using an extension spring balance

We feel the effects of forces whenever we take a ride on motor vehicles such as cars and motorcycles.

Dampers

Dampers are commonly used in vehicles such as motorcycles, 2. Take another two test tubes and label them ‘A’ and ‘B’ respectively. Add 3 cm3 of amylase
cars and trains. The spring in a damper (also called a shock solution into Test tube A and 3 cm3 of distilled water into Test tube B.
absorber) absorbs vibrations and tremors that occur when a
vehicle moves over uneven ground. However, the spring should 3. Add 3 cm3 of starch solution to each of the test tubes A, B and C. Shake the mixture in each
not vibrate too much and for too long after it is disturbed. test tube and allow the test tubes to stand for 20 minutes.
To make a motorcycle ride comfortable, a damper has to be
carefully designed by measuring (calibrating) how much force
the damper exerts when it is compressed. 4. After 20 minutes, add two drops of iodine solution into each test tube. Record the change in
colour of the iodine solution.

Test tube A:

Apparatus and Materials Test tube B:

• two identical spring balances • a piece of string 80 cm long Test tube C:


(calibrated in newtons) • six metallic objects of various masses, up to 500 g
• a retort stand • a rubber band

Discussion
Procedure and Observations
1. Which test tube shows that digestion of starch has occurred? Explain your answer.
Part A: Weights of Objects

1. Hang one of the spring balances on the retort stand.


Activities allow students
Use the string to attach the 500-g object to the hook
of the spring balance (Spring balance A).
Spring balance A
2. Name the product formed in Test tube A.
to practise interpreting
and analysing the data
Write down the mass of the 500-g object in kilograms.
3. Explain the purpose of Test tube B in this experiment.
Mass of object = kg 500g

What is the reading on Spring balance A?

Weight of 500-g object = N


retort stand

© 2014 Marshall Cavendish Education Pte Ltd Activity 11 13


collected, and to make
© 2014 Marshall Cavendish Education Pte Ltd Activity 14 29 logical conclusions.

Discussion and Extension include questions that


develop students’ ability to apply what they have learnt
from the activities to new situations. These questions
focus on the skills and processes of scientific inquir y,
including formulating hypotheses, posing questions,
and making inferences and predictions.

Extension
Isaac was pushing a rectangular box along the floor when he began to wonder if it would be
easier to push the box if he laid it on another side. To his surprise, he found that the task did
not seem easier or harder. He flipped the box onto its third side and still it seemed as easy as
before to move it.

He took a wooden box with all three sides different in area and laid it on the smallest side onto
the wooden plank. He slowly lifted up one end of the plank, tapping it to loosen the grip of the box
on the plank, to make it slide downwards. When the box finally started to slide down the slope,
Isaac measured the angle the plank made with the table.

wooden box

plank

He repeated the procedure with the side of the box with the largest surface area. He noted the
angle again. Then, he tested the last side of the box in the same way.

Isaac then drew the table below to record his measurements.

Angle at which box starts sliding


First angle Second angle Average
Smallest side of the box
Medium side of the box
Largest side of the box

(a) Suppose the force of friction depends on the angle at which the box starts sliding. What
possible hypothesis was Isaac testing?

(b) What results would not support the hypothesis stated in (a)? Explain your answer.

Questions marked with Optional for N(A)


are optional for Normal (Academic) students.
36 Chapter 14 © 2014 Marshall Cavendish Education Pte Ltd

Preface iii

iii-viii SS0281 (S)LSSPBB_Preface.indd iii 19/05/2014 16:27


CONTENTS
Preface iii
Safety Equipment in the Laborator y vi
Laborator y Apparatus vii

Theme Systems
Chapter 10 Transport System in Organisms
Activity 10.1 The Big Bouncy Quail Egg 1
Activity 10.2 Osmosis in Living Tissues 5
Activity 10.3 Make Your Own Coloured Carnation 9

Chapter 11 Human Digestive System


Activity 11.1 Digestion of Starch 11

Chapter 13 Electrical Systems


Activity 13.1 Setting Up Circuits 15
Activity 13.2 Measuring Electricity with Meters 19
Activity 13.3 Measuring Resistance 25

Theme Interactions
Chapter 14 Interactions Through the Application of Forces
Activity 14.1 Measuring Forces 29
Activity 14.2 Frictional Force 33
Activity 14.3 Magnetic Force 37
Activity 14.4 Pressure 41

Chapter 15 Energy and Work Done


Activity 15.1 Energy and Work Done 43
Activity 15.2 Energy Conversions 47
Activity 15.3 Source of Energy 51

iv Contents

iii-viii SS0281 (S)LSSPBB_Preface.indd iv 19/05/2014 16:31


Chapter 16 Transfer of Sound Energy Through Vibrations
Activity 16.1 Watching Sound Vibrations 53
Activity 16.2 Making Sound Optional for N(A) 57

Chapter 17 Effects of Heat and Its Transmission


Activity 17.1 Expansion of Solids and Liquids Due to Heat 61
Activity 17.2 Expansion of Gases Due to Heat 65
Activity 17.3 Investigating Applications of Heat Transfer 69
Activity 17.4 Convection Currents 73
Activity 17.5 Radiation of Heat 75

Chapter 18 Chemical Changes


Activity 18.1 Changes Caused by Mixing 77
Activity 18.2 Changes Caused by Heat and Light 79
Activity 18.3 Action of Acids on Metals and Carbonates 83
Activity 18.4 Properties of Acids 87
Activity 18.5 Properties of Alkalis 89

Chapter 19 Interactions Within Ecosystems


Activity 19.1 Effect of Light on Organisms 93
Activity 19.2 Effect of Acidity and Alkalinity on Organisms 97
Activity 19.3 Simulation of the Transfer of Energy in a Food Chain 101
Activity 19.4 Photosynthesis and Respiration 105

Contents v

iii-viii SS0281 (S)LSSPBB_Preface.indd v 19/05/2014 16:31


Safety Equipment in the Laboratory
Fire Extinguisher
There are two main types of fire extinguishers:
• carbon dioxide
• dry chemical (powder)

Fire Blanket
• Remove the blanket from the container.
• Wrap it around the victim to put out the fire.

Eye-wash Apparatus
• If an object gets in your eye, remove it from your eye immediately.
• Run fresh water over your eye for several minutes.

First Aid Kit


• Items in the kit may be used temporarily for small injuries.

Safety Shower
• Position yourself (or your laboratory partner) under the safety shower.
• Pull the handle to release the flow of water.
• Flames will be rapidly extinguished.

Empty Bucket
• The empty bucket can be used to contain water for putting out small fires.

Sand Bucket
• Dump the sand onto small fires to put them out.

vi Safety Equipment in the Laboratory

iii-viii SS0281 (S)LSSPBB_Preface.indd vi 19/05/2014 16:31


Laboratory Apparatus
Our senses help us to gather information around us. When we need to be more precise, we use
instruments that can help us to make accurate measurements and obtain exact figures.

You will learn to use many of these apparatus and equipment during your practical lessons.
Remember to use the respective apparatus and equipment carefully and correctly.

Apparatus Functions Apparatus Functions

Bunsen burner Retort stand

• To raise a flame
• To support
above the table
apparatus during
surface high
experiments
enough for heating

Heat

Tripod stand Test tube

• To contain small
• To support amounts of
apparatus during chemicals for
heating heating
• To mix solutions/
liquids

Round-bottomed flask Beaker

• To mix and heat • To contain


chemicals and chemicals
liquids evenly • To collect liquids

Safety Equipment
Laboratory
in the Laboratory
Apparatus vii

iii-viii SS0281 (S)LSSPBB_Preface.indd vii 19/05/2014 16:31


Apparatus Functions Apparatus Functions

Boiling tube Filter funnel


• To transfer liquid
• To contain into containers
chemicals for with a small
strong heating opening
• To collect and • To separate
hold liquids solids from
liquids

Conical flask Gas jar

• To contain and
mix chemicals • To collect gas
and liquids

Bell jar Evaporating dish

• To separate the • To evaporate


set-up of an the liquid in a
experiment from solution over a
its surroundings Bunsen burner

viii Safety Equipment


Laboratory Apparatus
in the Laboratory

iii-viii SS0281 (S)LSSPBB_Preface.indd viii 19/05/2014 16:32


Name: ( )

Chapter 10
Suggested Approx.
duration 2 days
Class: Date:

Activity
10.1 The Big Bouncy Quail Egg

Transport System in Organisms


Aim
• To observe the effect of osmosis

Apparatus and Materials


• a 100 cm3 beaker • a glass rod
• a raw quail egg • 50 cm3 of distilled water
• 150 cm3 of dilute hydrochloric acid

Procedure and Observations


Background Information:
Eggshell consists mainly of calcium carbonate. A shell membrane surrounds the quail’s egg white
and yolk. This membrane, which is partially permeable, is similar to a cell membrane.

1. Place a raw quail egg gently into the beaker.

2. Carefully pour 50 cm3 of dilute hydrochloric acid into the beaker. The acid must cover the
entire raw quail egg.

dilute hydrochloric acid


quail egg

3. Let the egg soak in the dilute hydrochloric acid for 10 minutes. Stir the acid occasionally.
Do this gently using a glass rod.

© 2014 Marshall Cavendish Education Pte Ltd Activity 10 1

001-010 SS0281 (S)LSSPBB_10.indd 1 19/05/2014 13:56


4. Record your observation of the quail egg.
There were bubbles and white foam on the surface of the eggshell. (Answers may vary.)

5. After 10 minutes, pour away the acid. Be careful not to pour the egg out.

6. Carefully rinse the egg with tap water. Examine the egg to see if there is still any shell material
on it. If there is, repeat steps 3 and 5.

7. In the table provided, record the longest length of the egg, its texture and its appearance.

longest length of egg

8. Fill the beaker with at least 50 cm3 of distilled water.

9. Place the egg gently into the beaker. Let the egg soak in the distilled water for 30 minutes.

10. After 30 minutes, remove the egg from the beaker.

11. Measure and record the length of the egg. Record the texture and appearance of the egg in
the table provided.

12. Place the egg back into the beaker of distilled water. Allow the egg to soak in the distilled water
for 24 hours.

13. After 24 hours, remove the egg from the beaker.

2 Chapter 10 © 2014 Marshall Cavendish Education Pte Ltd

001-010 SS0281 (S)LSSPBB_10.indd 2 19/05/2014 13:56


14. Measure and record the length of the egg. Record the texture and appearance of the egg in
the table provided.

Observations:

Texture and appearance


Length of egg (cm)
of egg

The texture is smooth and


Egg before submerging in
firm. The egg appears to be
distilled water translucent.

Egg after submerging in The texture is smooth and


distilled water for firm. The egg appears to be
30 minutes translucent.

The egg is firmer than before.


Egg after submerging in The egg appears to be very
distilled water for 24 hours translucent and swollen with
fluid.

(Answers vary.)

Discussion
1. Explain your observation of the egg when it was submerged in distilled water without its shell.
Water molecules moved from the distilled water into the egg. This is because distilled water has a

higher concentration of water molecules than the egg’s contents.

2. A raw quail’s egg with its shell removed is submerged in syrup. Predict what would be observed
after 24 hours. Explain your prediction.
The egg would shrink or shrivel, as water molecules would move from the egg into the syrup. This is

because syrup has a lower concentration of water molecules than the egg’s content.

3. Suggest another substance to replace hydrochloric acid.


Vinegar/Lemon juice/(Accept any other reasonable answer.)

© 2014 Marshall Cavendish Education Pte Ltd Activity 10 3

001-010 SS0281 (S)LSSPBB_10.indd 3 19/05/2014 13:56


Extension
1. A small drop of dye is added into a container of water. A raw egg with its shell removed is then
soaked in the coloured water. Predict what you would observe after 24 hours. Explain your
prediction.
The egg would swell and change in colour, depending on the colour of the water. This is because the

egg membrane is partially permeable. The egg membrane allows the coloured water to pass through it

and enter the egg by osmosis.

2. In markets, vegetables are often kept in the open for long hours.

(a) What undesirable effect would this condition have on the water content of vegetables?
The vegetables would lose most of their water from evaporation or for their own usage.

(b) From your knowledge, what type of solution might you spray on vegetables in the market
to reverse the undesirable effect? Explain your answer.
Water or distilled water. This is because water would enter the vegetables through osmosis.

This replenishes the water lost.

4 Chapter 10 © 2014 Marshall Cavendish Education Pte Ltd

001-010 SS0281 (S)LSSPBB_10.indd 4 19/05/2014 13:56


Name: ( )

Chapter 10
Suggested 2 periods
duration 70 minutes
Class: Date:

Activity
10.2 Osmosis in Living Tissues
Note for teachers:
You may want to conduct Activities 10.2 and 10.3 together since there is a long

Transport System in Organisms


waiting time for Activity 10.3.

Aims
• To investigate osmosis in living tissues
• To predict the changes to living cells that are immersed in solutions of different concentrations

Apparatus and Materials


• a razor blade/sharp scalpel • two petri dishes
• two onion rings that are at least • 30 cm3 of distilled water
50 mm in diameter • 30 cm3 of 20% sucrose solution

Note:
Place the onion rings in a plastic bag to prevent them from drying.

Procedure and Observations


1. Measure the diameter of the two onion rings as shown below. You may use the grid lines on
the next page to help in the measurement. Record the measurements in the table on the
next page.

diameter of onion ring

© 2014 Marshall Cavendish Education Pte Ltd Activity 10 5

001-010 SS0281 (S)LSSPBB_10.indd 5 19/05/2014 13:56


Grid lines for measurements

2. Place one onion ring in a petri dish containing distilled water. Place the other onion ring in a
petri dish containing concentrated sucrose solution.

3. After 20 minutes, remove the onion rings from the petri dishes. Measure the diameter of each
onion ring.

4. Record the measurements. Take note of the texture and appearance of each onion ring.
Record your observation in the table below.
Onion Change in
ring Initial diameter Final diameter Texture and
diameter
Solution (mm) (mm) appearance
(mm)

Distilled water

Concentrated
sucrose solution

(Answers vary.)

6 Chapter 10 © 2014 Marshall Cavendish Education Pte Ltd

001-010 SS0281 (S)LSSPBB_10.indd 6 19/05/2014 13:57


Discussion
1. What changes occurred in the diameter of the onion rings that were immersed in the distilled
water and concentrated sucrose solution? State the difference in the concentration of water
molecules in the onion cells and the solutions.

(a) Distilled water


The diameter of the onion ring increased. The onion cells have a lower concentration of water

molecules than the distilled water.

(b) Concentrated sucrose solution


The diameter of the onion ring decreased. The onion cells have a higher concentration of water

molecules than the concentrated sucrose solution.

2. Explain why the changes occurred in the onion rings immersed in:

(a) distilled water;


Water molecules moved from the distilled water into the onion cells. This caused the onion cells

to expand. Hence, the onion ring expanded and increased in diameter.

(b) concentrated sucrose solution.


Water molecules moved from the onion cells into the concentrated sucrose solution. This caused

the onion cells to shrink. Hence, the onion ring shrank and decreased in diameter.

© 2014 Marshall Cavendish Education Pte Ltd Activity 10 7

001-010 SS0281 (S)LSSPBB_10.indd 7 19/05/2014 13:57


Extension
Saltwater fish are usually found in places such as the ocean and saltwater ponds. As their name
suggests, these fish live in salt water. The bodies of saltwater fish are used to the high salt
content where most freshwater fish would not be able to sur vive.

(a) Predict what would occur if you tried to keep a tuna fish (a kind of saltwater fish) in a tank filled
with fresh water. Explain your prediction.
The tuna fish would die from having too much water in its body. This is because fresh water has a higher

concentration of water molecules than the body of the tuna fish. Hence, water molecules would move

from the fresh water into the body of the tuna fish.

(b) Salmon are able to live in both freshwater and saltwater environments. What are some
questions you need to ask in order to study this capability?
How are salmon different from freshwater fish?/How are salmon different from saltwater fish?/

What are the changes that occur when salmon travel between freshwater and saltwater environments?/

Is this capability temporary or permanent? (Accept any other reasonable answers.)

8 Chapter 10 © 2014 Marshall Cavendish Education Pte Ltd

001-010 SS0281 (S)LSSPBB_10.indd 8 19/05/2014 13:57


Name: ( )

Chapter 10
Suggested 2 periods
duration 70 minutes
Class: Date:

Activity
10.3 Make Your Own Coloured Carnation

Transport System in Organisms


Aim
• To investigate the movement of coloured dye up a flower stalk

Apparatus and Materials


• a 500 cm3 beaker • a white carnation flower with stalk and leaves
• 5 cm3 of red ink/food dye • a dropper
• 5 cm3 of green ink/food dye • 300 cm3 of distilled water
• 5 cm3 of blue ink/food dye • a scalpel

Procedure and Observations


1. Pour about 300 cm3 of distilled water into the beaker.

2. Using the dropper, add about 4 to 5 drops of ink/food dye of your choice into the beaker
of water.

Note:
To increase the colour intensity of the coloured water, add more drops of ink/food dye.

3. Cut the stalk of the carnation flower in water. Shorten the stalk to about 15 cm.

4. Place the flower stalk in the beaker of coloured water.

5. Place the beaker containing the flower stalk in a breezy place. After one hour, observe the
colour of the flower, stalk and leaves.

© 2014 Marshall Cavendish Education Pte Ltd Activity 10 9

001-010 SS0281 (S)LSSPBB_10.indd 9 19/05/2014 13:57


Discussion
1. State the plant tissue(s) involved in the transport of the ink particles and water molecules.
Xylem tissue

2. Explain how the water molecules moved up the flower stalk.


Water molecules moved up the stalk through osmosis. This is because the flower stalk has a lower

concentration of water molecules than the water in the beaker.

3. Why was the flower stalk placed in a breezy area? Explain your answer.
This is done to speed up the loss of water from the flower. This, in turn, would speed up the intake of

water by the flower.

4. Describe the appearance of the white carnation flower after one hour.
The white carnation flower changed to a colour that is the same as the water in the beaker.

5. Suggest another plant that can replace the carnation plant in this experiment. You must be
able to obtain visible results in this plant.
White lilies/White dahlia/(Accept any other reasonable answer.)

Extension
1. Suggest how you can create a two-coloured carnation flower.
First, fill two beakers with water of different colours. Arrange the two beakers side by side. Split the

stalk of a white carnation into two. Soak each end of the split stalk into the water in each beaker. After

a few hours, the carnation flower will appear to have two colours, one on each half of the flower.

2. Fertilisers contain essential substances for plant growth. Fertilisers come in liquid or solid
form. Plants can take in solid fertilisers only if the fertilisers are dissolved in water. However,
plants can take in liquid fertilisers directly. Explain why plants can take in liquid fertilisers
directly but not solid fertilisers.
Liquid fertilisers contain essential substances dissolved in water while solid fertilisers do not. The plant

transport system can only transport essential substances that are dissolved in a liquid. Hence, plants

can only take in liquid fertilisers directly but not solid fertilisers.

10 Chapter 10 © 2014 Marshall Cavendish Education Pte Ltd

001-010 SS0281 (S)LSSPBB_10.indd 10 19/05/2014 13:57


Name: ( )

Chapter 11
Suggested 1 period
duration 35 minutes
Class: Date:

Activity
11.1 Digestion of Starch

Human Digestive System


Aims
• To test for starch using iodine solution
• To investigate the action of amylase on starch

Apparatus and Materials


• five test tubes • a test-tube rack
• a test-tube holder • a marker pen
• three 5 cm3 syringes • a bottle of dilute iodine solution
• a beaker containing 20 cm3 • a beaker containing 20 cm3
of 1% starch solution of 1% amylase solution
• a beaker containing 20 cm3
of distilled water

Note:
Two pupils should share the apparatus and materials.

Procedure and Observations


Background Information:
The flour that we use for baking bread contains a large amount of starch. When we bake bread,
we add amylase to the dough. As a result, the starch in flour is broken down into simpler forms of
sugar, such as glucose and maltose. We also add yeast to the dough. Yeast converts the simpler
forms of sugar into alcohol and carbon dioxide in a chemical process called fermentation. When
bread is baked, the products of fermentation allow the bread to rise and give bread its taste.

Yeast is able to produce its own amylase to break down the starch in flour. However, yeast takes a
long time to do so. Hence, adding amylase speeds up the process of baking bread.

© 2014 Marshall Cavendish Education Pte Ltd Activity 11 11

011-014 (S)LSSPBB_11.indd 11 19/05/2014 13:58


Part A: Test for Starch

1. Using a syringe, add 2 cm3 of starch solution into a test tube.

2. Add two drops of iodine solution to the starch solution as shown in the diagram below.

iodine solution
solution

starch solution
starch solution

3. Record the change in colour of the iodine solution.


The colour of the iodine solution changes from brown to blue-black.

4. Using a new syringe, add 2 cm3 of distilled water into another test tube.

Note:
The variable in this experiment is the type of test substance (starch and distilled water). The
volume of starch and distilled water must be kept constant for a fair comparison. Hence, the test
tube with 2 cm3 of distilled water is the control set-up.

5. Add two drops of iodine solution to the distilled water.

6. Record the change in colour of the iodine solution.


The colour of the iodine solution remains brown.

Discussion
What can you infer from the results obtained?
Iodine changes in colour from brown to blue-black in the presence of starch.

12 Chapter 11 © 2014 Marshall Cavendish Education Pte Ltd

011-014 (S)LSSPBB_11.indd 12 19/05/2014 13:58


Part B: Action of Amylase on Starch

1. Add 3 cm3 of amylase solution into a test tube. Boil the solution gently for three minutes and
cool it under a tap. Label this test tube ‘C’.
water

2. Take another two test tubes and label them ‘A’ and ‘B’ respectively. Add 3 cm3 of amylase
solution into Test tube A and 3 cm3 of distilled water into Test tube B.

3. Add 3 cm3 of starch solution to each of the test tubes A, B and C. Shake the mixture in each
test tube and allow the test tubes to stand for 20 minutes.

4. After 20 minutes, add two drops of iodine solution into each test tube. Record the change in
colour of the iodine solution.

Test tube A: The colour does not change.

Test tube B: The colour of the iodine solution changes from brown to blue-black.

Test tube C: The colour of the iodine solution changes from brown to blue-black.

Discussion
1. Which test tube shows that digestion of starch has occurred? Explain your answer.
Test tube A. The colour of the iodine solution remained brown, showing the absence of starch.

2. Name the product formed in Test tube A.


Maltose

3. Explain the purpose of Test tube B in this experiment.


It serves as a control to show that starch is not broken down on its own during this experiment.

© 2014 Marshall Cavendish Education Pte Ltd Activity 11 13

011-014 (S)LSSPBB_11.indd 13 19/05/2014 14:01


4. You were told to mix the contents of Test tube A and Test tube C. Predict the colour of the
mixture after 20 minutes. Explain your prediction.
The mixture would be brown in colour. This is because the amylase that was in Test tube B would digest

the starch in Test tube C. Thus, there would be an absence or very little starch after 20 minutes.

5. Temperature affects how proteins work. From your observations of Test tube C, what can you
infer about enzymes?
Enzymes are proteins as their ability to work is affected by temperature.

Extension
1. Which organs involved in the digestion of food produce amylase?
Salivary glands and pancreas

2. What can you infer about the product formed in Part B, Test tube A?
Iodine does not change in colour in the presence of maltose.

3. If test tubes A and C produced the same results in Part B, what are some probable reasons
for this?
The amylase in Test tube C was not boiled long enough./The amylase was not boiled throughout.

(Accept any other reasonable answer.)

4. Five drops of concentrated hydrochloric acid was added to a test tube containing 3 cm3
amylase, and 3 cm3 starch solution. After 20 minutes, two drops of iodine solution were
added to the test tube. The iodine colour changed from brown to blue-black. What can you infer
from this?
Hydrochloric acid prevented starch from being broken down.

14 Chapter 11 © 2014 Marshall Cavendish Education Pte Ltd

011-014 (S)LSSPBB_11.indd 14 19/05/2014 14:01


Name: ( )

Chapter 13
Suggested 1 period
duration 35 minutes
Class: Date:

Activity
13.1
Setting Up Circuits

Electrical Systems
Aims
• To set up electrical circuits according to circuit diagrams
• To investigate the effects of varying the resistance on the brightness of the bulb

We set up closed electrical circuits whenever we connect a USB cable to a computer.

Universal Serial Bus (USB) Connections

Most computer devices today are attached to computers


through USB connections. A USB connection allows data
to be transferred between a device and the computer.
The USB connection works by allowing electric current
to flow between the device and the computer.

Devices that use the USB connection include the


computer mouse, keyboard, printer, and speaker.
The USB connection is also commonly used to power and
charge the batteries of mobile phones and many other
kinds of portable electronic devices.

Apparatus and Materials


• a dry cell (1.5 V) • a light bulb (1.5 V or higher)
• two connecting wires with crocodile clips • a switch
• 30 cm of nichrome/constantan wire

© 2014 Marshall Cavendish Education Pte Ltd Activity 13 15

015-028 SS0284 (S)LSSPBB_13.indd 15 19/05/2014 14:02


Procedure and Observations

Part A: Identifying Circuit Symbols

1. Observe the electrical circuit set up on your teacher’s bench.

2. In the space provided, draw a circuit diagram to show how the circuit on your teacher’s bench is
connected.

Part B: Connecting a Circuit

1. Connect the components to form a circuit. The light bulb in your circuit should light up.
(Be careful when handling the nichrome wire which may be hot.)

cell

light bulb
nichrome wire

16 Chapter 13 © 2014 Marshall Cavendish Education Pte Ltd

015-028 SS0284 (S)LSSPBB_13.indd 16 19/05/2014 14:03


2. Adjust the crocodile clip so that only 1 cm of the nichrome wire is connected to the circuit.
Obser ve the brightness of the light bulb.

cell
crocodile clips

light bulb 1 cm

nichrome wire

3. Move the crocodile clips further apart until about 20 cm of the nichrome wire is connected to
the circuit.

cell

light bulb 20 cm
nichrome wire

What do you observe about the brightness of the light bulb when a longer part of the nichrome
wire is connected to the circuit?
The light bulb becomes dimmer.

4. Disconnect the nichrome wire from the circuit. Connect the clips to the second light bulb. Draw a
diagram of the new circuit in the space below.

(Accept any reasonable answer.)

What do you observe about the brightness of the first light bulb when the second light bulb
is connected?
The light bulb becomes dimmer compared to the light bulb in Step 2.

© 2014 Marshall Cavendish Education Pte Ltd Activity 13 17

015-028 SS0284 (S)LSSPBB_13.indd 17 19/05/2014 14:03


Discussion
1. Why does the light bulb become dimmer when a longer part of the nichrome wire is connected?
The longer nichrome wire increases the resistance of the circuit. This causes the current in the circuit

to decrease. Hence, the bulb becomes dimmer.

2. In Step 4, another light bulb replaces the nichrome wire. Based on your observations, what
can you infer about the circuit resistance when a circuit component is connected in series?
Connecting a light bulb in series increases the circuit resistance.

18 Chapter 13 © 2014 Marshall Cavendish Education Pte Ltd

015-028 SS0284 (S)LSSPBB_13.indd 18 19/05/2014 14:03


Name: ( )

Chapter 13
Suggested 1 period
duration 35 minutes
Class: Date:

Activity
13.2
Measuring Electricity with Meters

Electrical Systems
Aim
• To learn how to connect an ammeter and a voltmeter to a circuit

We can connect electrical measuring instruments to test common household electrical components.

Multimeters

Multimeters are convenient tools that electricians use for


measuring current, voltage, and resistance. Instead of
using separate instruments (ammeters and voltmeters),
a multimeter can switch between measuring current and
voltage by the turn of a knob.

In a household electrical circuit, multimeters can be used


to test different types of fuses, batteries and incandescent
light bulbs.

Apparatus and Materials


• a dry cell (1.5 V) • a switch
• four connecting wires with crocodile clips • a voltmeter
• a light bulb (1.5 V) • an ammeter

© 2014 Marshall Cavendish Education Pte Ltd Activity 13 19

015-028 SS0284 (S)LSSPBB_13.indd 19 19/05/2014 14:03


Procedure and Observations

Part A: Connecting the Ammeter

1. Set up the circuit as shown. Use the switch to close the circuit.

1 2
0 3
A

Draw the corresponding circuit diagram in the space below.

2. Record the reading on the ammeter (Ammeter reading 1).


Ammeter reading 1 = (Answers vary.)

3. Disconnect the bulb and ammeter from the circuit. Exchange their positions in the circuit
as shown.

1 2
0 3
A

20 Chapter 13 © 2014 Marshall Cavendish Education Pte Ltd

015-028 SS0284 (S)LSSPBB_13.indd 20 19/05/2014 14:03


Draw the corresponding circuit diagram in the space below.

4. Record the new reading on the ammeter (Ammeter reading 2).


Ammeter reading 2 = (Answer same as in Step 2)

Discussion
1. Compare Ammeter reading 1 with Ammeter reading 2. What can you infer about the electric
current flowing through components connected in series in a circuit?
The same current flows through components connected in series.

2. After conducting this experiment, Marie thinks that the positions of the dry cell and switch
in Step 1 of Part A could also be exchanged without affecting the ammeter reading. She
connects them as shown in the diagram below. (Do not connect this circuit.)

Predict how the arrangement of components in Marie’s circuit will affect the ammeter reading.
Explain your prediction.
The needle will deflect left. The positive terminal of the ammeter is connected to the negative terminal of

the cell. The current in the ammeter thus flows in the wrong direction.

© 2014 Marshall Cavendish Education Pte Ltd Activity 13 21

015-028 SS0284 (S)LSSPBB_13.indd 21 19/05/2014 14:04


Part B: Connecting the Voltmeter

1. Connect the voltmeter in parallel to the light bulb.

1 2
0 3
A

1 2
0 3
V

Draw the corresponding circuit diagram in the space below.

2. Record the reading on the voltmeter and ammeter (Ammeter reading 3).

Voltmeter reading = (Answers vary.)

Ammeter reading 3 = (Answers vary.)

Does the ammeter reading change when you connect the voltmeter? No

22 Chapter 13 © 2014 Marshall Cavendish Education Pte Ltd

015-028 SS0284 (S)LSSPBB_13.indd 22 20/5/14 8:24 AM


3. Now, disconnect the switch and voltmeter from the circuit. Connect the voltmeter in place of the
switch, according to the diagram shown below.

1 2 1 2
0 3 0 3
V A

Draw the corresponding circuit diagram in the space below.

V A

4. Record the new reading on the ammeter (Ammeter reading 4).

Ammeter reading 4 = OA

Does the bulb light up? No

© 2014 Marshall Cavendish Education Pte Ltd Activity 13 23

015-028 SS0284 (S)LSSPBB_13.indd 23 20/5/14 8:24 AM


Discussion
1. Compare Ammeter reading 3 with Ammeter reading 4. What can you infer about the resistance
of a voltmeter?
No current (or a very small current) flows in the circuit when the voltmeter is connected in series.

This shows that the resistance of the voltmeter is very high.

2. In the circuit diagram below, the voltmeter is connected in parallel with the switch.

Predict the voltmeter reading. Explain your prediction.


The voltmeter reading would be very small, or almost zero. The switch is a good conductor and has a

very low resistance. Very little energy is needed to move charges through the switch.

24 Chapter 13 © 2014 Marshall Cavendish Education Pte Ltd

015-028 SS0284 (S)LSSPBB_13.indd 24 19/05/2014 14:04


Name: ( )

Chapter 13
Suggested 2 periods
duration 70 minutes
Class: Date:

Activity
13.3
Measuring Resistance

Electrical Systems
Aim
• To find out the resistance (R) across a lamp by measuring the potential difference (V) across it and
the current (I) through it

We can use rheostats to control the loudness of individual sounds produced by sound systems.

Sound Mixers

Disc jockeys (or DJs) use audio or sound mixers to play


music or mix sounds at parties. Each of the many sliders
and knobs on a sound mixer is a rheostat that controls the
resistance of an individual electrical circuit in the mixer.

When the DJ moves a slider down or turns a knob


anti-clockwise, the resistance of a circuit is increased.
This decreases the current, causing the sound produced
by that circuit to become softer.

Apparatus and Materials


• a dry cell (1.5 V) • a rheostat
• seven connecting wires with crocodile • a voltmeter
clips • an ammeter
• a light bulb (1.5 V)
• a switch

© 2014 Marshall Cavendish Education Pte Ltd Activity 13 25

015-028 SS0284 (S)LSSPBB_13.indd 25 19/05/2014 14:04


Procedure and Observations

Part A: Connecting the Rheostat

1. Take the rheostat and observe the slider and its three terminals. On the diagram below, label
the slider with the letter S, and the terminals with the letter T.
S

T T

2. Place the rheostat on your bench, as shown in the diagram above. Then, move the slider all
the way to the right.

3. Connect the negative terminal (–) of the cell to the single terminal on the left of the rheostat.
Connect another crocodile clip to the upper terminal on the right of the rheostat.

to negative crocodile clip


terminal (–) of cell

crocodile clip

Discussion
Does current pass through the upper terminal on the right of the rheostat, or through the lower
terminal on the right?
The upper terminal

Part B: Investigating the Relationship Between V and I

1. Set up the circuit as shown below. Check that the positive and negative terminals of the
ammeter and voltmeter are correctly connected.

1 2
0 3
A

1 2
0 3
V

26 Chapter 13 © 2014 Marshall Cavendish Education Pte Ltd

015-028 SS0284 (S)LSSPBB_13.indd 26 19/05/2014 14:05


2. Draw the corresponding circuit diagram in the space below.

3. Close the circuit and read the ammeter and voltmeter. Record your readings in the table below.
Calculate the ratio of V to I.

4. Move the rheostat slider about a few centimetres to the left. Continue doing this until you
observe that the ammeter reading changes by at least 0.1 A.

Record the new voltmeter and ammeter readings in the table below. Calculate the new ratio of
V to I. Record this value in the same table.

5. Repeat Step 4 until you have recorded five sets of readings.

Potential difference V (V) Current I (A) V ÷ I (Ω)

(Answers vary.)

© 2014 Marshall Cavendish Education Pte Ltd Activity 13 27

015-028 SS0284 (S)LSSPBB_13.indd 27 19/05/2014 14:05


Discussion
1. The ammeter reading changes when the slider moves from the right to the left. What can you
infer about circuit resistance and the position of the slider?
The circuit resistance decreases when the slider is moved towards the left.

2. Based on your calculations of the ratio of V to I, what can you infer about the resistance of
the light bulb during the experiment?
The resistance (ratio of V to I) stays constant for smaller values of current and then it increases when

current increases.

Extension

A B

Marie wants to show that the resistance of the rheostat is maximum when only terminals A and B
of the rheostat above are connected to a circuit. By drawing the direction of the current flow, explain
briefly why this is so.
When the wires are connected to terminals A and B, the current flows through a longer section of the coil.

The resistance of a conductor increases with its length. Hence, the coil has maximum resistance when the

wires are connected to terminals A and B.

28 Chapter 13 © 2014 Marshall Cavendish Education Pte Ltd

015-028 SS0284 (S)LSSPBB_13.indd 28 19/05/2014 14:06


Name: ( )

Chapter 14
Suggested 2 periods
duration 70 minutes
Class: Date:

Activity
14.1 Measuring Forces

Application of Forces
Interactions Through the
Aim
• To measure forces using an extension spring balance

We feel the effects of forces whenever we take a ride on motor vehicles such as cars and motorcycles.

Dampers

Dampers are commonly used in vehicles such as motorcycles,


cars and trains. The spring in a damper (also called a shock
absorber) absorbs vibrations and tremors that occur when a
vehicle moves over uneven ground. However, the spring should
not vibrate too much and for too long after it is disturbed.
To make a motorcycle ride comfortable, a damper has to be
carefully designed by measuring (calibrating) how much force
the damper exerts when it is compressed.

Apparatus and Materials


• two identical spring balances • a piece of string 80 cm long
(calibrated in newtons) • six metallic objects of various masses, up to 500 g
• a retort stand • a rubber band

Procedure and Observations

Part A: Weights of Objects

1. Hang one of the spring balances on the retort stand.


Use the string to attach the 500-g object to the hook
of the spring balance (Spring balance A).
Spring balance A
Write down the mass of the 500-g object in kilograms.

Mass of object = 0.5 kg 500g

What is the reading on Spring balance A? retort stand

Weight of 500-g object = 5 N

© 2014 Marshall Cavendish Education Pte Ltd Activity 14 29

029-042 SS0281 (S)LSSPBB_14.indd 29 19/05/2014 14:07


2. Repeat Step 1 using the other objects that are given to you. Record the mass and weight of
each object in the table below.

Mass of object (kg) Weight (N)

Discussion
Compare the mass and weight you recorded in Step 2. How are they related?
The weight in newtons is about 10 times the value of the mass in kilograms.

Part B: Contact Forces on a Rubber Band

1. Remove Spring balance A from the retort stand and attach it to the rubber band.

2. Hold Spring balance A with one hand. Using your other hand, stretch the rubber band to twice
its length. (Do this on the horizontal surface of the laboratory bench. Use the metre rule to
check the length of the rubber band.)
Spring balance A rubber band

How much force is your hand pulling on the rubber band with?

Pulling force of hand on rubber band = N (Answers vary.)

3. Attach the other end of the rubber band to the second spring balance (Spring balance B).
Stretch the rubber band by pulling on both spring balances. Check that the stretched length of
the rubber band is the same as in Step 2.

Spring balance A rubber band Spring balance B

Record the readings on both spring balances below.

Force measured by Spring balance A = N

Force measured by Spring balance B = N (Answers vary.)

30 Chapter 14 © 2014 Marshall Cavendish Education Pte Ltd

029-042 SS0281 (S)LSSPBB_14.indd 30 19/05/2014 14:07


Discussion
1. Spring balance A in Step 2 measures the force that the left hand exerts on the rubber band.
Similarly, what force does Spring balance B measure in Step 3?
It measures the force that the right hand exerts on the rubber band.

2. Based on your answer in the previous question, what can you say about the sizes of the two
opposite forces that stretch a rubber band?
The forces that stretch a rubber band in opposite directions are equal.

Extension
1. An astronaut carries out Part A of this experiment on the moon. Why are the readings of the
weight different from the experiment done on earth?
The moon’s gravity is weaker than earth’s. Hence, the downward force applied on each object would

be smaller.

2. Suppose a second spring balance (Spring balance C) was hung below the spring balance in
Part A.

Which spring balance would give the same reading as the


spring balance in Part A of the experiment? Explain.
Spring balance C. Spring balance A would measure the combined Spring
balance A
weight of Spring balance C and the 500-g weight. Only Spring

balance C measures the weight of the 500-g weight itself.

retort stand Spring


balance C

500g

© 2014 Marshall Cavendish Education Pte Ltd Activity 14 31

029-042 SS0281 (S)LSSPBB_14.indd 31 19/05/2014 14:07


3. Alice modifies the apparatus in Part B of this experiment. She repeats Step 3 of Part B by
replacing the rubber band with various other objects. Predict whether the reading on Spring
balance A will be equal to the reading on Spring balance B when Alice replaces the rubber
band with:

(a) a spring;
Yes, the readings on both balances will be equal.

(b) a rigid metal ring;


Yes, the readings on both balances will be equal.

(c) a third spring balance.


Yes, the readings on both balances will be equal.

4. Isaac hooks two spring balances together and then pulls on them. The two balances do not move.

Predict if the readings of the two balances are equal.


Yes, they are equal.

32 Chapter 14 © 2014 Marshall Cavendish Education Pte Ltd

029-042 SS0281 (S)LSSPBB_14.indd 32 19/05/2014 14:08


Name: ( )

Chapter 14
Suggested 2 periods
duration 70 minutes
Class: Date:

Activity
14.2 Frictional Force

Application of Forces
Interactions Through the
Aim
• To investigate the effect of friction on different objects

Apparatus and Materials


• a coin • a rubber eraser
• a spring balance • a metre rule
• 30 cm of string • three identical, heavy books
• a small rectangular wooden block • 8 to 10 round pencils

Procedure and Observations

Part A: Effect of Friction on Different Objects

1. Place the coin, the wooden block and the eraser on the metre rule. Each object should be
roughly 20 cm apart from each other.

eraser

wooden block

coin
20 ¢

2. Gently raise the end of the metre rule that is nearer to the eraser.

3. Observe the objects as you raise the rule higher. Which object moves first? Write down the
order in which each object moves.
The coin moves first, followed by the wooden block, then the eraser.

© 2014 Marshall Cavendish Education Pte Ltd Activity 14 33

029-042 SS0281 (S)LSSPBB_14.indd 33 19/05/2014 14:08


Discussion
1. Based on your observation in Step 3, which object has the roughest surface? Explain.
The rubber eraser has the roughest surface. It slips down the wooden rule last. Hence, the frictional

force between the rubber and the wood is highest.

2. Would your observation change if the positions of the objects were interchanged with each
other? Give a reason for your answer.
No. As the metre rule is raised, the surface roughness of the objects and the ruler remain unchanged.

The downward force (weight) pulling on each object also remains unchanged. Hence, the objects start

sliding in the same sequential order.

Part B: Effect of Friction on a Stack of Books

1. Place one book on the table, insert a loop of string between two pages in the middle of the
book, as shown. Pull gently and horizontally on the string with a spring balance.

2. When the book moves slowly and smoothly at a constant speed, record the reading on the
spring balance. Do this three times, then calculate the average reading. Record your readings
in the table given on the next page.

3. Repeat steps 1 and 2 using three books instead of one book.

34 Chapter 14 © 2014 Marshall Cavendish Education Pte Ltd

029-042 SS0281 (S)LSSPBB_14.indd 34 19/05/2014 14:08


4. Repeat Step 3, using 8 to 10 pencils placed under the three books.

Three books
Measured force (N) One book
No pencils pencils

First reading

Second reading

Third reading

Average reading

(Answers vary.)

Discussion
1. Compare the average force exerted on the books in Step 2 with the average force exerted in
Step 3. Explain your obser vation.
The force exerted in Step 3 is larger than the force in Step 2. The weight of the three books is larger

and increases the friction with the bench surface.

2. Compare the average force exerted on the books in Step 3 with the average force exerted in
Step 4. Explain your observation.
The force exerted in Step 3 is larger than the force in Step 4. The pencils reduce the friction between

the books and the bench.

© 2014 Marshall Cavendish Education Pte Ltd Activity 14 35

029-042 SS0281 (S)LSSPBB_14.indd 35 19/05/2014 14:08


Extension
Isaac was pushing a rectangular box along the floor when he began to wonder if it would be
easier to push the box if he laid it on another side. To his surprise, he found that the task did
not seem easier or harder. He flipped the box onto its third side and still it seemed as easy as
before to move it.

He took a wooden box with all three sides different in area and laid it on the smallest side onto
the wooden plank. He slowly lifted up one end of the plank, tapping it to loosen the grip of the box
on the plank, to make it slide downwards. When the box finally started to slide down the slope,
Isaac measured the angle the plank made with the table.

wooden box

plank

He repeated the procedure with the side of the box with the largest surface area. He noted the
angle again. Then, he tested the last side of the box in the same way.

Isaac then drew the table below to record his measurements.

Angle at which box starts sliding


First angle Second angle Average
Smallest side of the box
Medium side of the box
Largest side of the box

(a) Suppose the force of friction depends on the angle at which the box starts sliding. What
possible hypothesis was Isaac testing?
The force of friction between the box and plank depends on the surface area on which the box rests

on the plank.

(b) What results would not support the hypothesis stated in (a)? Explain your answer.
The averages of the measured angles remain approximately constant when the box rests on sur faces

of different areas. This result would show that the force of friction remains constant when the resting

area changes.

36 Chapter 14 © 2014 Marshall Cavendish Education Pte Ltd

029-042 SS0281 (S)LSSPBB_14.indd 36 19/05/2014 14:08


Name: ( )

Chapter 14
Suggested 1 period
duration 35 minutes
Class: Date:

Activity
14.3 Magnetic Force

Application of Forces
Interactions Through the
Aim
• To investigate the effects of magnetic force

We can obser ve magnetic forces all around us.

Maglev Trains

Magnetic levitation (or Maglev) trains are


trains that can reach ver y high speeds of
travel due to electromagnets lifting the train
slightly above the tracks. This lifting (levitation)
reduces friction between the train and the
railway tracks. Magnetic forces also drive the
train for ward and slow it down.

Many Maglev train networks are found in


Japan, Germany and China. Maglev trains can
travel at speeds of up to 500 km/h.

Apparatus and Materials


• two strong bar magnets • an electronic balance
• two toy cars with free-moving wheels • a metre rule
• a roll of adhesive tape

© 2014 Marshall Cavendish Education Pte Ltd Activity 14 37

029-042 SS0281 (S)LSSPBB_14.indd 37 19/05/2014 14:09


Procedure and Observations

Part A: Magnets on Toy Cars

1. Set up the magnets on the two toy cars as shown in the diagram below.
magnets
adhesive tape adhesive tape
S N N S

Bring the two cars as closely together as possible. Then, release the toy cars.

2. Using the metre rule, measure the distance between the final positions of the two toy cars.
adhesive tape magnets adhesive tape
S N N S

Distance between toy cars = cm (Answers vary.)

Part B: Measuring the Magnetic Force of Repulsion

1. Let the lower magnet lie on its side on the scale pan as shown. Attach a very small piece of
adhesive tape to secure the magnet to the pan of the balance.

N S

2. Take the reading (X1) of the balance.

38 Chapter 14 © 2014 Marshall Cavendish Education Pte Ltd

029-042 SS0281 (S)LSSPBB_14.indd 38 19/05/2014 16:40


3. Hold the other magnet horizontally above it so that the like poles are facing each other.

N S

N S

4. Advance the magnet very slowly downwards and towards the other magnet.

5. Take the last reading (X2) of the electronic balance just before the magnets come into contact.

6. Record your readings in the table below.

Readings on electronic
Difference in readings (g) Repulsive force (N)
balance (g)
X2 – X1
X1 X2 X2 – X1 × 10
1000

Average repulsive force (N)

(Answers vary.)

7. Repeat steps 3 to 6 two more times and calculate the average repulsive force.

8. What assumption have you made in measuring the force exerted?


The adhesive tape contributes a negligible weight to the final reading on the electronic balance.

© 2014 Marshall Cavendish Education Pte Ltd Activity 14 39

029-042 SS0281 (S)LSSPBB_14.indd 39 19/05/2014 14:09


Extension
1. Isaac has a collection of toy cars of different sizes, masses and designs. He repeats steps 1
and 2 in Part A of the experiment, using two different cars each time. He finds that the distance
in Step 2 is different for each type of toy car.

Based on his findings, Isaac concludes that the force between the pair of toy cars depends
on the size, mass and design of the toy cars. Do the results of his experiment support
his conclusion? Explain.
No. There are other factors affecting the distance that he measured in Step 2. Isaac should directly

measure the force between the magnets using spring balances.

2. Ivy is planning an experiment to find out how the size of two magnets affects the repulsive
force between them.

What questions should Ivy ask before conducting her experiment?


How do the shapes of the magnets affect the repulsive force?/To determine the size of each magnet,

what measurements must I make?

40 Chapter 14 © 2014 Marshall Cavendish Education Pte Ltd

029-042 SS0281 (S)LSSPBB_14.indd 40 19/05/2014 14:10


Name: ( )

Chapter 14
Suggested 2 periods
duration 70 minutes
Class: Date:

Activity
14.4 Pressure

Application of Forces
Interactions Through the
Aim
• To investigate pressure using the formula: pressure = force / area

We can make use of the effect of atmospheric pressure to pick up large objects.

Suction Lifter

Suction lifters are useful tools for handling fragile


pieces of glass. A suction lifter works very much like
a suction hook in our bathrooms and kitchens. A lever
is pushed to remove some air from the two suction
cups, creating a partial vacuum in the cups. The higher
air pressure outside pushes the pane of glass firmly
against the cups.

A suction lifter allows a worker to pick up glass panes


and marble tiles easily. Compared to our fingers, a
suction lifter holds on to objects more firmly.

Apparatus and Materials


• two to three sheets of bubble wrap (A4 size) • a mass scale (up to 5 kg)
• a thin piece of wood, about 4 cm × 4 cm • a 30-cm ruler

Procedure and Observations


1. Cut out a square piece of bubble wrap such that it contains piece of
one bubble. bubble wrap
bubble d
Measure the length of one side of the bubble wrap. Calculate
the area of the bubble wrap and record the value below.

Area of bubble wrap = d 2 = cm2

2. Place the piece of bubble wrap on the mass scale. Press on the bubble wrap with your thumb.
Gently increase the pressure on the bubble until the bubble bursts.

© 2014 Marshall Cavendish Education Pte Ltd Activity 14 41

029-042 SS0281 (S)LSSPBB_14.indd 41 19/05/2014 14:10


3. Record the reading on the scale, y, at the moment of the bubble bursting, in the table provided.
Then, calculate the force exerted to burst the bubble. Record this value in the same table.

Reading Reading on scale, y (kg) Force exerted to burst bubble, y × 10 (N)


1
2
3
Average
(Answers vary.)

4. Repeat steps 1 to 3 with two other pieces of bubble wrap.

5. Calculate the average value of y. Then, calculate the average force exerted to burst a bubble.

6. Calculate the pressure needed to burst the bubble.


Average force on bubble
Pressure on bubble = = = N / cm2
Area of bubble wrap

7. Repeat steps 1 to 6 using two larger pieces of bubble wrap. Each piece of bubble wrap
should have two or more bubbles on it. (You may need to use the square piece of wood to
help you press on the bubbles evenly.)

Record your results in the table below.

Average force needed to Pressure needed to burst


Area of bubble wrap (cm2)
burst bubbles (N) bubbles (N/cm2)

(Answers vary.)

Discussion
1. From your observations, what can you conclude about the area of the wrap and the force
needed to burst the bubbles on it?
The larger the area, the larger the force needed to burst the bubbles.

2. The pressure needed to burst one bubble is in fact larger than that calculated in Step 6.
Suggest an explanation for this.
The wrap is larger than the bubble. Hence, the force is in fact exerted over a smaller area.

42 Chapter 14 © 2014 Marshall Cavendish Education Pte Ltd

029-042 SS0281 (S)LSSPBB_14.indd 42 19/05/2014 14:10


Name: ( )

Chapter 15
Suggested 1 period
duration 35 minutes
Class: Date:

Activity
15.1 Energy and Work Done

Energy and Work Done


Aim
• To investigate the different types of energy changes

Apparatus and Materials


• two identical steel wires • a retort stand with clamp
• a spring • a 100-g mass

Procedure and Observations

Part A: Steel Wire

1. Take a steel wire and bend it in the middle five to six times.

2. Carefully touch the middle of the wire. How does the middle of the wire feel?
It feels warm.

3. Repeat Step 1 using the second steel wire. Bend the wire in the middle five to six times more
gently and slowly.

4. Carefully touch the middle of this wire. How does the middle of the wire feel?
It feels warmer, but cooler than the bent wire in Step 2.

© 2014 Marshall Cavendish Education Pte Ltd Activity 15 43

043-052 SS0281 (S)LSSPBB_15.indd 43 19/05/2014 14:13


Discussion
1. Explain whether work is done on the wire when you bend it.
Work is done on the wire. My hands apply forces on the wire. Each half of the wire moves in the direction of

the force.

2. What energy conversions take place when you bend the wire?
Chemical energy from the body → Kinetic energy of the wire → Heat

3. Compare your observations in steps 2 and 4. What can you infer about work done on the wire
in these two steps?
The first wire feels warmer because more work is done on the first wire in the same amount of time. Hence,

more kinetic energy is converted into heat.

Part B: Spring and Mass

1. Hang the spring from the retort stand.

spring

100-g mass

2. Attach the 100-g mass to the free end of the spring.

3. Pull the 100-g mass down by 2 cm. What happens to the spring?
It stretches.

4. Release the spring. What do you observe?


The spring at first contracts, then extends. The mass vibrates up and down.

44 Chapter 15 © 2014 Marshall Cavendish Education Pte Ltd

043-052 SS0281 (S)LSSPBB_15.indd 44 19/05/2014 14:13


Discussion
1. Explain whether work is done on the mass when you pull the 100-g mass.
Work is done on the spring. My hand applies a downward force on the mass. The mass moves in the

direction of this force.

2. When you release your fingers from the spring, is work done on the mass? Explain your
answer.
Yes, work is done on the mass. The spring applies an upward force on the mass, which moves upwards in

the direction of the force.

3. When you release your fingers, the spring coils upwards. What energy conversions are taking
place when this happens?
Elastic potential energy → Kinetic energy + Gravitational potential energy

Extension
1. A plastic ruler is bent in the same way as the wire in Part A. However, the ruler does not feel hot.
Suggest two reasons for this.
Firstly, plastic is a poor conductor of heat. Any heat generated in the ruler is not conducted quickly to

the hands. Secondly, in contrast to the stiff wire, less work is done on the flexible plastic material. This is

because less force is needed to bend (move) the ruler.

2. The metal of the spring in Part B bends and unbends when the spring vibrates up and down.
Unlike the wire, why does the spring not feel warm?
The vibrating spring does not bend as much or bend as quickly as the wire. Most of the work done on the

spring is converted to elastic potential energy rather than heat.

3. Isaac wants to find out if a spring produces heat when it is stretched. What are some questions
that Isaac needs to ask in order to plan and carry out this experiment?
How can the heat produced by the spring be measured?/Should the string be stretched by a mass that

vibrates from one of its ends?/By how much should the spring be extended?

© 2014 Marshall Cavendish Education Pte Ltd Activity 15 45

043-052 SS0281 (S)LSSPBB_15.indd 45 19/05/2014 14:14


4. A solar cooker makes use of the sun’s energy to fry eggs. Such a cooker can be made by lining
the inner curved surface of a small wok with many small mirrors. A tripod stand can be used
to set up a small frying pan above the wok. If the cooker is placed in strong, direct sunlight,
the concentrated heat from the sun can be used to fry eggs.
ray
so
f su
nlig
ht

frying pan
egg

plane
mirrors

wok
rocks plasticine
large tin

Besides frying eggs, solar cookers can also be used to boil water into steam.

What other modifications would you suggest to the set-up described above, in order to convert
the sun’s energy into other forms of useful energy, e.g. electrical energy? (Hint: Refer to
Activity 1.4, Uses and Power of Steam, in Practical Book Vol. A.)
In place of a frying pan on the tripod stand, set up a round-bottomed flask filled with water. Channel the

boiling water into a jet of escaping steam. Direct the jet of steam at a turbine connected to an electrical

generator. The set-up hence converts the sun’s energy into electrical energy.

46 Chapter 15 © 2014 Marshall Cavendish Education Pte Ltd

043-052 SS0281 (S)LSSPBB_15.indd 46 20/5/14 8:28 AM


Name: ( )

Chapter 15
Suggested 1 period
duration 35 minutes
Class: Date:

Activity
15.2
Energy Conversions

Energy and Work Done


Aim
• To investigate the energy conversions in a pendulum

Energy wastage in moving vehicles can be better managed by energy conversion technology.

Flywheel Energy Storage

Ordinar y brake systems rely on friction to slow


down cars and trains. The heat produced in this
process wastes energy. A technology called
Flywheel Energy Storage (FES) can reduce energy
wastage. When a moving train slows down and
stops at a station, the kinetic energy is transferred
to a free-spinning wheel. The stationar y train is
started up again by releasing the kinetic energy
from the spinning flywheel. The FES thus allows
the train’s kinetic energy to be stored and re-used.

Today, the FES is used in some roller coaster


systems and as well as in motor-racing vehicles.

Apparatus and Materials


• a pendulum bob (diameter 1 cm) • a block of wood (2 cm thick)
• a retort stand fixed with two clamps • two metre-rules
• a piece of string (about a metre long)

© 2014 Marshall Cavendish Education Pte Ltd Activity 15 47

043-052 SS0281 (S)LSSPBB_15.indd 47 19/05/2014 14:14


Procedure and Observations
1. Clamp the metre rule to the retort stand. The metre rule should stand vertically on the
bench surface.

2. Hang the pendulum bob from the second (lower) clamp on the retort stand. Adjust the
pendulum bob so that it hangs just above the sur face of the bench.

metre rule

first clamp

second clamp

pendulum bob

wooden block

3. Place the wooden block on the bench so that the pendulum bob is just touching it. Mark the
position of the wooden block on the bench.

4. Pull the pendulum bob away from the block by a height, h. Measure h using the metre rule.
Release the bob to let it swing and collide with the wooden block. The wooden block should
slide in a straight line. (You may need to try this a few times.)

metre rule

wooden
bob block
s

metre rule

5. Use the other metre rule to measure the distance, s, travelled by the block on the bench.

6. Repeat steps 3 to 5 using different values of h.

48 Chapter 15 © 2014 Marshall Cavendish Education Pte Ltd

043-052 SS0281 (S)LSSPBB_15.indd 48 19/05/2014 14:14


7. Record your measurements in the table below.

Height of pendulum bob, h (cm) Distance moved by wooden block, s (cm)

(Answers vary.)

Discussion
1. (a) From the measurements, we can conclude that as the height at which the bob was released
(h) increases, the distance moved by the wooden block (s) increases .

(b) Provide a possible explanation of why this is so.


The higher the pendulum bob is displaced, the greater its gravitational potential energy. This

greater amount of energy is converted into a greater amount of kinetic energy as the pendulum

swings to the bottom of the arc. The energy is used to do a greater amount of work on the wooden

block, hence causing the block to slide a further distance on the bench.

2. State the energy conversion that occurs when the pendulum bob swings from the highest point
to the lowest point (just before it collides with the block).
Gravitational potential energy → Kinetic energy

3. Is work done on the block when the pendulum bob collides with it? Explain your answer.
The pendulum bob applies a force on the block very briefly. The block moves in the same direction as

the force. Hence, work is done on the block during the moment of contact.

4. What happens to the kinetic energy of the block as it slides on the bench?
The kinetic energy is converted to heat and sound energy.

5. Is work done on the block when it slides on the bench? Explain your answer.
Since the block’s speed decreases, a force (friction) is acting on the block. The block moves in a direction

parallel but opposite to the force. Work is done on the block to convert its kinetic energy to heat and sound.

© 2014 Marshall Cavendish Education Pte Ltd Activity 15 49

043-052 SS0281 (S)LSSPBB_15.indd 49 19/05/2014 14:15


Extension
1. Tim observes that if the surface bench is smooth, the wooden block will slide further. He
sprinkles some powder on the bench surface and repeats the procedure.

(a) In what other ways can Tim make the block slide further?
Rub the surface of the block with sandpaper to smoothen it./Use a block of smaller mass./Use

a pendulum bob of greater mass.

(b) Tim wants to convert as much of the potential energy of the pendulum as possible to
the kinetic energy of the block. He tries using different materials for the pendulum bob
and the block. How can he tell from his obser vations whether he has succeeded?
He hears very little sound during each collision./The pendulum bob bounces back very little after

each collision.

2. Newton’s cradle is a toy found in many offices and homes. Like a simple pendulum, it can
be used to demonstrate conser vation of energy. The toy is made of five or more metallic
pendulum bobs. The toy is set in motion by raising one or more of the bobs to one side. The
swinging bob or bobs collide with the stationar y bobs and cause them to swing to the other
side.

(a) The balls eventually stop swinging after some time. Does this show that energy is not
conserved? Explain why.
No, the total energy is always conserved. During each collision, some kinetic energy is converted to

sound energy and heat. Eventually, all the kinetic energy is converted. Hence, the balls stop swinging.

(b) How can the toy be made to move for as long as possible? Suggest at least one method of
doing so.
Let the toy run in a bell jar connected to a pump. Use the pump to remove all the air from the jar.

(Accept other possible answers.)

50 Chapter 15 © 2014 Marshall Cavendish Education Pte Ltd

043-052 SS0281 (S)LSSPBB_15.indd 50 19/05/2014 14:15


Name: ( )

Chapter 15
Suggested 1 period
duration 35 minutes
Class: Date:

Activity
15.3
Source of Energy

Energy and Work Done


Aim
• To investigate how electrical energy is generated using a moving magnet

The muscles in our body can supply energy to light up electric torchlights.

Human-powered Electric Torchlights

The muscles in our body can be used as an energy source


for electric torchlights. A generator is connected to the
light bulb of the torchlight. The user can either shake the
torchlight or turn a handle to rotate a small generator.
The motion causes a permanent magnet in the torchlight
to move near a coil of wire. This generates the electrical
energy needed to light up the bulb.

Human-powered torchlights are very useful in emergencies. These torchlights


were used during the Second World War. This is because batteries were often
scarce and power stations were unreliable then.

Apparatus and Materials


• a large coil of wire (solenoid), 15 cm long • a pair of connecting wires
• a very strong magnet • a galvanometer

Procedure and Observations


1. Use the connecting wires to connect the ends of the coil of wire (solenoid) to the galvanometer.

2. Point one pole of the magnet towards one end of the coil of wire. The magnet should be about
10 cm away from the coil of wire.

© 2014 Marshall Cavendish Education Pte Ltd Activity 15 51

043-052 SS0281 (S)LSSPBB_15.indd 51 19/05/2014 14:15


3. Move the magnet horizontally into the solenoid.

galvanometer
magnet

solenoid

What do you observe on the galvanometer?


The galvanometer needle deflects (or moves to the left/right).

4. Now pull the magnet out from the coil. What do you observe on the galvanometer?
The needle deflects in the opposite direction (or moves to the right/left).

5. If you move the magnet faster in and out of the coil, what do you observe?

The needle will deflect more.

Discussion
The galvanometer needle moves when electric current flows in the coil of wire. What energy
conversions take place when you move the magnet into the coil of wire?
Kinetic energy (of the moving magnet) → Electrical energy in the coil of wire

Extension
Instead of moving the magnet into and out of a stationary coil of wire, Isaac tries to make the
magnet stationary and move only the coil.

Predict if Isaac will observe any change in the size and direction of the galvanometer’s deflection.
Suggest an explanation for this.
No, the magnitude and direction of the deflection is unchanged. The electric current generated in the coil

depends only on the motion of the magnet relative to the coil and vice versa.

52 Chapter 15 © 2014 Marshall Cavendish Education Pte Ltd

043-052 SS0281 (S)LSSPBB_15.indd 52 19/05/2014 14:15


Name: ( )

Chapter 16
Suggested 2 periods
duration 70 minutes
Class: Date:

Activity
16.1
Watching Sound Vibrations

Through Vibrations
Transfer of Sound Energy
Aim
• To investigate the effects of sound energy through vibrations

Apparatus and Materials


• a balloon • a radio
• one roll of cling film • half a teaspoon of rice or salt
• a baking tray • a wooden spoon
• a toothpick • two identical wine glasses

Procedure and Observations

Part A: Travelling Sound Vibrations

1. Place the bowl on a table. Stretch the cling film tightly over the top of the bowl.

2. Sprinkle some rice or salt on the cling film.

3. Hold the baking tray over the bowl.

4. Use the wooden spoon to hit the tray hard. This will create a loud noise.

© 2014 Marshall Cavendish Education Pte Ltd Activity 16 53

053-060 SS0281 (S)LSSPBB_16.indd 53 19/05/2014 14:16


5. What happens to the grains of rice or salt on the cling film as you hit the tray?
The rice or salt jumps as the tray is hit.

Discussion
What caused the particles of rice or salt to behave in this way?
The loud sound produced by the wooden spoon hitting the tray.

Part B: Sound Vibrations from a Radio

1. Inflate the balloon by blowing into it.

2. Turn on the radio and place your hands around the balloon. Hold the balloon about 10 cm away
from the radio.

How does the balloon feel?


Very slight vibrations can be felt from the balloon./ No vibration is felt from the balloon.

3. Turn up the volume on the radio and repeat Step 2. How does the balloon feel?
Stronger vibrations can be felt from the balloon.

Discussion
1. Which part of the balloon in Part B is vibrating in response to the sounds from the radio?
The inflated rubber material of the balloon.

54 Chapter 16 © 2014 Marshall Cavendish Education Pte Ltd

053-060 SS0281 (S)LSSPBB_16.indd 54 19/05/2014 14:16


2. (a) Notice that the rice on the cling film and the balloon itself were at a distance from the
source of the sound (the tray and the radio respectively). Yet sound was transmitted to
them. How do you think this came about?
The banging of the tray and the vibrations of the radio’s speakers caused the air molecules next

to them to vibrate. The energy of these vibrating molecules is passed on to the surrounding

molecules until it reaches the cling film (on the bowl) and the balloon. This energy is transmitted

as sound energy.

(b) Suggest a reason why rice grains were sprinkled on the cling film.
The visible movement of the rice grains shows very clearly that the cling film vibrates.

3. Why does an increase in the volume of the radio cause a change in the vibration of the balloon?
The sound energy from the radio is increased when it is louder. This extra energy is transferred to the

air molecules and passed on to the molecules on the balloon’s skin. This causes the balloon’s skin

to vibrate with greater energy too.

Part C: Moving Objects Using Vibrating Wine Glasses

1. Put a little amount of water into a wine glass.

2. Hold the wine glass steady at the base with one hand.

3. Wet a finger of the other hand using water from the wine glass.

4. Rub the rim of the wine glass with your wet finger and listen to the
sound produced. Do not rub the rim of the wine glass too hard or too fast.

5. Fill the other identical wine glass with the same amount of water.
Listen to the sound produced.

6. Add or pour away water to adjust the pitch of the sound, so


that it matches that of the first wine glass.

7. Place the two wine glasses close to each other but not
touching each other. Balance a toothpick on the rim of the
second wine glass.

8. Rub the rim of the first wine glass to produce the sound. What happens to the toothpick?
The toothpick moves.

© 2014 Marshall Cavendish Education Pte Ltd Activity 16 55

053-060 SS0281 (S)LSSPBB_16.indd 55 19/05/2014 14:16


Discussion
1. What is the material that vibrates to produce the sound when you rubbed the wine glass with
your finger?
The glass

2. What can you infer from the observation of the toothpick as you rubbed the wine glass?
The vibrations of the first wine glass are transmitted by air as sound to the second wine glass.

Extension
1. Loud noises can cause damage to our eardrums. Based on your answer in Part B Question 3
when the volume of the radio is turned up, infer why this is so.
Loud noises are carried to the eardrums by molecules with very high energy. It might even exceed

the threshold of pain. The eardrum takes in the high energy of these vibrations and begins to vibrate

so vigorously that it can even burst. The cling film and the balloon show us how a skin can pick up

vibrations easily like the eardrum.

2. The National Environment Agency (NEA) of Singapore, which regulates noise levels in our
environment, does not allow sounds louder than 60 dB (decibels) during the day at certain
places. What are some examples of places that restrict excessive noise levels?

Schools, hospitals, homes for the elderly, residential areas

56 Chapter 16 © 2014 Marshall Cavendish Education Pte Ltd

053-060 SS0281 (S)LSSPBB_16.indd 56 19/05/2014 14:16


Name: ( )

Chapter 16
Suggested 2 periods
duration 70 minutes
Class: Date:

Optional for N(A)


Activity
16.2 Making Sound

Through Vibrations
Transfer of Sound Energy
Aims
• To produce sounds of different frequency
• To investigate the relationship between frequency and pitch

Human beings, unlike other organisms, can hear sounds only within a certain range of frequencies.

Dog Whistle

Not all whistles produce sounds that we can hear. Dog


whistles or silent whistles produce sounds that are of
extremely high frequencies. The human ear cannot pick up
the sounds produced by dog whistles.

However, dogs can hear these sounds. This is because


the frequency range of a dog whistle is within the range of
hearing of dogs. The sounds produced by these whistles do
not disturb anyone. These whistles can also capture a dog’s
attention quickly as such high-pitched sounds are uncommon.
Dog whistles are mainly used in the training of dogs.

Apparatus and Materials


• a set of tuning forks • a retort stand
• a pail of water • a rubber bung or rubber pad
• 1 m long tube (glass, PVC,
bamboo or aluminium),
at least 3 cm in diameter

© 2014 Marshall Cavendish Education Pte Ltd Activity 16 57

053-060 SS0281 (S)LSSPBB_16.indd 57 19/05/2014 14:17


Procedure and Observations

Part A: The Resonance Tube

1. Dip a short length of the 1 m long tube in a bucket of water. Keep the tube in an upright
position.

resonance tube

air L

water

2. Blow across the top of the tube and note the sound that is produced. Keep in mind the sound
that you just heard. (Try humming the note to yourself softly.)

3. Lower the tube a little way into the water and blow across the top again. How is the pitch of
the sound different from the sound you heard in Step 2?
The pitch is now higher.

4. Lower the tube a little more and blow again. How is the pitch of the sound different from the
sound you heard in Step 3?
The pitch is now even higher.

Discussion
1. What is vibrating in order to produce the sound?
The column of air in the tube above the water

2. Suggest a possible hypothesis about the length (L) of air in the tube and the frequency you hear.
The shorter the column of air in the tube, the higher the frequency with which it vibrates.

58 Chapter 16 © 2014 Marshall Cavendish Education Pte Ltd

053-060 SS0281 (S)LSSPBB_16.indd 58 19/05/2014 14:17


Part B: Tuning Fork

1. Observe the length of the prongs of the tuning fork.

number printed on tuning


fork here

Write down the number printed on the tuning fork. This number is the frequency of the sound
produced by the tuning fork.
256 Hz (Answer may vary.)

2. Hold the tuning fork by the stem and strike it smartly on the rubber bung, or on a rubber pad
on the table. Hold the tuning fork 5 to 10 cm from your ear.

Can you hear any sound from the tuning fork? No

3. Strike the tuning fork again and hold its stem against a tabletop.

Can you hear any sound from the tuning fork? Yes

4. Why can you hear the sound from the fork only when the stem of the tuning fork is resting on
a tabletop?
The table picks up the vibrations of the tuning fork and vibrates with the same frequency. It has a larger

surface so vibrations from the table are passed on to surrounding air molecules.

5. Select another tuning fork. Write down the number printed on this tuning fork.
284 Hz (Answers vary.)

6. Is the frequency you wrote down in step 5 higher or lower than the first tuning fork?
Higher/Lower (Answers vary.)

7. Repeat Step 3 using the second tuning fork. Is the pitch of the sound produced by this fork
higher or lower?
Higher/Lower (Answers vary.)

© 2014 Marshall Cavendish Education Pte Ltd Activity 16 59

053-060 SS0281 (S)LSSPBB_16.indd 59 19/05/2014 14:17


Discussion
1. Which part of the tuning fork vibrates when it is struck?

The prongs of the fork

2. How is the pitch of a tuning fork related to the length of its prongs?
The longer the prongs, the lower the pitch.

Part C (Optional): Demonstrating Resonance

1. Hum the note of the tuning fork that your teacher has just struck and is pressing onto the table.

2. Raise and lower the resonance tube, and blow into it until the same note is heard in the tube
as the note you are humming. Keep the tube in this position with the help of a retort stand.

3. tuning fork

resonance tube

Strike the tuning fork on the rubber bung again, and hold the prongs of the fork above the
mouth of the tube. A loud sound should be heard. (If you do not hear a loud sound, adjust the
position of the tube and repeat this step.)

Discussion
1. The air in the tube is vibrating to produce the sound. What is causing the air in the tube to vibrate?
The tuning fork

2. The sound produced has the same frequency as the tuning fork. What is the frequency at
which the air in the tube vibrates?
(Frequency of the tuning fork chosen, in hertz.)

60 Chapter 16 © 2014 Marshall Cavendish Education Pte Ltd

053-060 SS0281 (S)LSSPBB_16.indd 60 19/05/2014 14:17


Name: ( )

Chapter 17
Suggested 2 periods
duration 70 minutes
Class: Date:

Activity
17.1 Expansion of Solids and Liquids Due to Heat

Its Transmission
Effects of Heat and
Aim
• To investigate the effects of heat on solids and liquids

The expansion and contraction due to heat can affect the accuracy of scientific obser vations.

Effects of Heat on Astronomical Telescopes

Telescopes are often used outdoors on cold nights.


The cooled mirrors and lenses in telescopes contract and
may become misaligned. When the system of lenses and
mirrors fails to transmit or reflect light rays correctly, the
images formed become distorted or out of focus.

Telescope makers therefore carefully consider the effects


of heat on mirrors and lenses. Astronomers who use large
telescopes also take special care to keep the mirrors and
lenses at the right temperatures.

Apparatus and Materials


Part A
• a set of ball and ring apparatus • a Bunsen burner
• a fireproof mat • a lighter
• a pair of tongs

Part B
• a boiling tube • a 1,000 cm3 beaker
• a kettle of hot water (about 70°C) • 500 cm3 of tap water
• a retort stand and clamp • a stirrer
• a rubber stopper with capillary tube • a marker pen
• 15 to 20 ice cubes • 500 cm3 of water coloured with potassium
manganate(VII)

© 2014 Marshall Cavendish Education Pte Ltd Activity 17 61

061-076 SS0281 (S)LSSPBB_17.indd 61 19/05/2014 14:18


Procedure and Observations

Part A: Thermal Expansion of Solids

1. Take a set of the ball and ring apparatus. Hold the ball by
the chain and try to pass it through the ring.
ring
What do you observe?
ball
The ball goes through the ring easily.

2. Light the Bunsen burner. Using the tongs, hold the ball by the chain. Then heat the ball over the
flame for one to two minutes.

tongs

3. Suggest a hypothesis that is being tested in the experiment.


The metal ball expands when heated.

4. Remove the Bunsen burner and try to pass the hot ball through the ring again. Can the
ball now pass through the ring?

No

5. After 20 minutes, try to pass the ball through the ring again. Can the ball pass through the ring?

Yes

62 Chapter 17 © 2014 Marshall Cavendish Education Pte Ltd

061-076 SS0281 (S)LSSPBB_17.indd 62 19/05/2014 14:18


Discussion
1. Explain the observations made in steps 4 and 5.
The metal ball expanded when it was heated. Therefore, it was unable to pass through the ring. When

the ball cooled to room temperature, it contracted back to its original size. Hence, it was able to pass

through the ring again.

2. Suppose that in Step 1, instead of heating the ball, you cooled the ring in ice for a few minutes.
Predict if the ball would be able to pass through the ring.
No, the ball would be unable to pass through the ring.

Part B: Thermal Expansion of Liquids

1. Fill the boiling tube to the brim with the coloured water. Insert the stopper with the capillary tube
carefully. Do this over the sink to avoid wetting the bench. Ensure that there are no air bubbles
in the capillary tube so that some of the coloured water will rise up the capillary tube above the
level of the rubber stopper.

2. Half-fill the beaker with tap water. Clamp the boiling tube to the retort stand. Adjust the retort
stand to lower the tube into the beaker of water. In the diagram below, mark the level of the
coloured water in the capillary tube.

capillary tube

rubber stopper

coloured water
boiling tube
water

(Answers vary.)

© 2014 Marshall Cavendish Education Pte Ltd Activity 17 63

061-076 SS0281 (S)LSSPBB_17.indd 63 19/05/2014 14:18


3. Add ice into the beaker and stir the mixture. Observe the level of the coloured water in the capillary
tube. In the diagram below, mark the level of the coloured water in the capillary tube.

ice

coloured water
cold water
(Answers vary.)

(If the water level drops beyond the bottom of the capillary tube, readjust the initial level of
the coloured water. This will ensure that the water level stays within the capillary tube.)

4. Remove the boiling tube from the beaker. Pour away the mixture of cold water and ice. Fill
the beaker with hot water from the kettle. Then, immerse the boiling tube in the hot water.
Observe the water level in the capillary tube.

What do you observe about the water level in the capillary tube?
The water level falls slightly before rising.

5. The position of the water level in the capillary tube eventually comes to a rest. Mark the final
position of the water level in the tube in the diagram below.

coloured water
hot water

(Answers vary.)

Discussion

Explain your observation of the water level in the capillary tube in Step 4 (when you first immersed
the boiling tube in hot water).
The water level initially fell when the boiling tube was immersed in hot water. This is because heat travelled

to the boiling tube first. The boiling tube expanded and caused the water level to fall.

64 Chapter 17 © 2014 Marshall Cavendish Education Pte Ltd

061-076 SS0281 (S)LSSPBB_17.indd 64 19/05/2014 14:18


Name: ( )

Chapter 17
Suggested 2 periods
duration 70 minutes
Class: Date:

Activity
17.2 Expansion of Gases Due to Heat

Its Transmission
Effects of Heat and
Aim
• To investigate the effect of heat on gases

Apparatus and Materials


• a round-bottomed flask • a large beaker
• a beaker of coloured water • a rubber stopper fitted with a capillary tube
• a kettle of hot water at around 70°C • 500 cm3 of tap water
• a retort stand and clamp • a marker pen

Procedure and Observations


1. Take the rubber stopper fitted with a capillary tube. Dip one end of the capillary tube in the
coloured water so that about 1 cm of the end is immersed.

rubber stopper

about 1 cm capillary tube

coloured water

2. Cover the other end of the capillary tube with your thumb and lift the tube out of the water. The
length of the coloured water trapped in the tube should be 5 to 8 mm. Hold the tube horizontally.
coloured water 5 to 8 mm long

horizontal capillary tube

© 2014 Marshall Cavendish Education Pte Ltd Activity 17 65

061-076 SS0281 (S)LSSPBB_17.indd 65 19/05/2014 14:19


3. Tilt the tube slightly and remove your thumb from its end briefly. Let the coloured thread of water
(index) slide, little by little, towards the middle of the capillary tube.
coloured water index

horizontal capillary tube

4. When the index has slid half the distance towards the stopper, use your finger to cover the end of
the tube that was dipped into the coloured water. This stops the liquid index from sliding further.

5. Keeping your finger at the end of the tube, insert the rubber stopper into the round-bottomed
flask. Then, release your finger. The liquid index has now trapped a volume of air within the
round-bottomed flask.

6. Clamp the round-bottomed flask to the retort stand. Then, lower it into a large beaker of tap
water. In the diagram below, mark the position of the coloured index on the capillary tube.

tap water

(Answers vary.)

7. Add ice to the tap water and stir the mixture. Wait two to three minutes, then write down what
you observe about the position of the coloured index in the tube.
The index falls down the capillary tube.

8. In the diagram below, mark the position of the liquid index when it has stopped moving.

ice cubes

ice water

(Answers vary.)

66 Chapter 17 © 2014 Marshall Cavendish Education Pte Ltd

061-076 SS0281 (S)LSSPBB_17.indd 66 19/05/2014 14:19


9. Next, discard the cold water and ice in the beaker. Hold the flask in the palms of your hands and
observe the index. What do you observe?
The index rises.

10. Fill the beaker with hot water and immerse the flask into the hot water. Observe the liquid
index to check if it rises or falls within the next few seconds. What do you observe?
It falls a little before rising steadily up the tube.

11. When the index has stopped moving, mark its position on the diagram below.

hot water

(Answers vary.)

Discussion
1. Why did the liquid index behave as it did when the flask was immersed in ice water in Step 8?
The air in the tube contracted as the temperature fell, causing the index to fall.

2. Why did the liquid index move when you held the bottom of the flask in your hands? What does
this show?
My warm hands heated up the air in the flask. The expanding air pushed the liquid index up the tube. The

large response of the liquid index shows that a gas expands a lot when it absorbs a small amount of heat.

3. When the flask was plunged into hot water, why did the liquid index drop at first, and then rise?
The hot water heated up and expanded the flask first, which increased the volume of the flask slightly.

Since the air had not yet expanded, the weight of the liquid index pushed the air slightly down into the extra

volume. The air, which was heated up eventually, expanded and pushed the liquid index back up the tube.

© 2014 Marshall Cavendish Education Pte Ltd Activity 17 67

061-076 SS0281 (S)LSSPBB_17.indd 67 19/05/2014 14:19


4. State the conclusion of this experiment. Is this conclusion applicable to solids and liquids?
Gases expand when heated and contract when cooled. This conclusion is also applicable to both

solids and liquids.

Extension
Based on your obser vations in the experiment, explain why it is unsafe to store gas tanks that
are full in the event of a fire.
A full gas tank contains gas pressure at or near the maximum safe pressure. In the event of a fire, the gas

is heated and expands, increasing the pressure. If the gas pressure increases beyond the safe limit the

tanks may explode when the tank cannot withstand the high pressure.

68 Chapter 17 © 2014 Marshall Cavendish Education Pte Ltd

061-076 SS0281 (S)LSSPBB_17.indd 68 19/05/2014 14:19


Name: ( )

Chapter 17
Suggested 2 periods
duration 70 minutes
Class: Date:

Activity
17.3 Investigating Applications of Heat Transfer

Its Transmission
Effects of Heat and
Aim
• To investigate how different solids expand at different rates

Apparatus and Materials


• a bimetallic strip (brass and steel) • a pair of tongs
• a Bunsen burner • a lighter
• a marker pen

Procedure and Observations


1. Sketch the bimetallic strip given to you in the space provided below. Label the metals in the
drawing. Your teacher will tell you the names of the metals that form the strip.

(Answers vary.)

2. Light the Bunsen burner. Using a pair of tongs, hold the strip over the flame. Move the strip
steadily back and forth over the flame and turn over the strip from time to time. This ensures
that the whole length of the strip is heated evenly on both sides.

Heat the strip for one to two minutes and observe its behaviour. What happens to the strip as
it is being heated?

It bends.
bimetallic strip

© 2014 Marshall Cavendish Education Pte Ltd Activity 17 69

061-076 SS0281 (S)LSSPBB_17.indd 69 19/05/2014 14:19


3. In the space provided below, sketch the shape of the bimetallic strip after heating. Label the
name of the metal for each strip.

(Answers vary.)

4. Place the strip in the ice compartment of the laborator y refrigerator. After five to ten minutes,
remove it from the refrigerator. What has happened to the shape of the strip?
The strip is bent in the opposite direction.

Discussion
1. Describe how the bimetallic strip appears to have been made.
It is made from two different metals riveted together.

2. Based on the results of the experiment, suggest the hypothesis that is being tested in this
experiment.
Different metals expand by different amounts/at different rates when heated by the same amount.

3. Based on your obser vations in Step 3, which metal expands more when the bimetallic strip
is heated?
Brass expands more than steel.

4. The strip changes shape after it is heated. How does this tell you which metal expands more?
The strip which expands more is on the outside of the strip when it bends.

70 Chapter 17 © 2014 Marshall Cavendish Education Pte Ltd

061-076 SS0281 (S)LSSPBB_17.indd 70 19/05/2014 14:19


5. When the bimetallic strip is cooled, which metal contracts more?
Brass

6. The strip changes shape after it is cooled. How does this tell you which metal contracts more?
The strip which contracts more is on the inside of the strip when it bends.

Extension
1. Name at least two household appliances in which the bimetallic strip might be used.
Ovens, fridges, toasters, rice cookers, water heaters, air-conditioners

2. The bimetallic strip changes shape quickly when heated. How is this important for one of the
appliances you named in Question 1?
This is important in ovens because the bimetallic strip has to respond quickly to changes in temperature

in order to switch off the heating element before the food gets overcooked.

3. (a) A bent bimetallic strip is made by riveting a steel strip to a brass strip that is slightly
shorter. Predict what happens to the bimetallic strip when it is heated.
brass
steel

The bimetallic strip straightens/becomes less bent.

(b) A long, unbent strip made of the same metals as above is bent into a tight coil at room
temperature. A long metallic pointer is fixed to the centre of the coil.

steel

brass

(i) The coil is now heated. Predict in which direction the pointer will turn.
Anti-clockwise

© 2014 Marshall Cavendish Education Pte Ltd Activity 17 71

061-076 SS0281 (S)LSSPBB_17.indd 71 19/05/2014 14:20


(ii) Another coil is made with the steel and brass interchanged, as shown below.

brass

steel

Predict in which direction the pointer will turn.


Clockwise

72 Chapter 17 © 2014 Marshall Cavendish Education Pte Ltd

061-076 SS0281 (S)LSSPBB_17.indd 72 19/05/2014 14:20


Name: ( )

Chapter 17
1 – 2 periods
Suggested
35 – 70
duration minutes
Class: Date:

Activity
17.4
Convection Currents

Its Transmission
Effects of Heat and
Aim
• To investigate convection

We can feel the effect of convection currents deep in the earth in the form of earthquakes.

Convection Currents of Lava

The movement of the ground that we call


earthquakes is caused by convection currents
deep inside the earth (the mantle). The liquid mantle
magma (molten rock) nearer the centre of the
earth is heated and rises. The cooler liquid
magma near the top sinks. The moving liquid
outer core
pushes the plates of the earth’s crust. When
the plates rub against each other, the ground
under our feet moves and jerks and we feel an inner core
earthquake.

Apparatus and Materials


• a glass tube • a tripod stand
• wire gauze • a Bunsen burner
• a 250 cm3 beaker, • a lighter
filled with tap water • three to five small crystals of potassium
manganate(VII)

Procedure and Observations

1. Place the beaker of water on the wire gauze on the tripod stand.

2. Use the glass tube to slide one or two crystals of potassium manganate(VII) into the water.
Allow the crystals to sink to the bottom of the beaker.

3. Light the Bunsen burner to heat the water. Observe the movement of the purple streaks as the
potassium manganate(VII) crystal dissolves in the water.

© 2014 Marshall Cavendish Education Pte Ltd Activity 17 73

061-076 SS0281 (S)LSSPBB_17.indd 73 19/05/2014 14:20


4. By drawing arrows in the diagram below, sketch the convection current that you have observed
in Step 3.

beaker of water

crystals of potassium
manganate(VII)

wire gauze

tripod stand

(Answers vary.)

Discussion
What is the purpose of using the cr ystals of potassium manganate(VII) in the experiment?
The crystals allow us to observe the movements of the current clearly due to the colour of the crystals.

Extension
List some examples of the role of convection currents in nature.
Convection currents play a role in the movement of sea breezes and ocean currents.

74 Chapter 17 © 2014 Marshall Cavendish Education Pte Ltd

061-076 SS0281 (S)LSSPBB_17.indd 74 19/05/2014 14:20


Name: ( )

Chapter 17
Suggested 1 period
duration 35 minutes
Class: Date:

Activity
17.5 Radiation of Heat

Its Transmission
Effects of Heat and
Aim
• To investigate how the nature of a surface can affect absorption of heat radiation

Apparatus and Materials


• a tin can, shiny and silver • a shiny lid with hole, stopper and thermometer
• a tin can, dull and painted black • a black lid with hole, stopper and thermometer
• a Bunsen burner • wire gauze
• a lighter • a metre rule
• two polystyrene squares

Procedure and Observations


1. Set up the apparatus as shown in the diagram below. Place the tin cans on polystyrene
squares. Each can should be about 10 cm away from a Bunsen burner wrapped in wire gauze.

thermometer

Bunsen burner
wrapped in wire
gauze

air
air
tin
10 cm 10 cm tin

Dull, black surface Shiny, silver surface

Record the temperature readings of the two thermometers in the table given in Step 2.

© 2014 Marshall Cavendish Education Pte Ltd Activity 17 75

061-076 SS0281 (S)LSSPBB_17.indd 75 19/05/2014 14:20


2. Light the Bunsen burner. Wait eight minutes and switch off the burner. Record the temperature
readings of the two thermometers (after absorption).

Dull, black can Shiny, silver can

Temperature before absorption (°C)

Temperature after absorption (°C)

Rise in temperature (°C)


(Answers vary.)

Discussion
1. What is the purpose of the wire gauze?
Heat can be radiated from a hot surface. The Bunsen burner is not an object with a surface, so it heats

the gauze first, and the gauze gives out the radiant heat.

2. Which tin can is the better absorber of heat? The black and dull one

76 Chapter 17 © 2014 Marshall Cavendish Education Pte Ltd

061-076 SS0281 (S)LSSPBB_17.indd 76 19/05/2014 14:21


Name: ( )

Chapter 18
Suggested 1 period
duration 35 minutes
Class: Date:

Activity
18.1 Changes Caused by Mixing

Chemical Changes
Aim
• To observe the chemical changes caused by mixing different substances

Chemical changes caused by mixing can be obser ved in ever yday life.

Hair Dyes

Hair colouring is a practice that dates back to ancient


times. During a conventional process of hair colouring, the
outer layer of hair is opened using the alkaline solution,
ammonia. Chemicals in the hair dye undergo reactions to
remove colour from the hair. At the same time, the colour
present in the hair dye penetrates the inner portion of the
hair. This changes the colour of the hair.

The use of hair dyes may produce negative side effects.


Chemicals found in hair dyes may react with the skin,
causing irritation and allergy.

Apparatus and Materials


• a small glass bottle with a fitting cork • a spatula
• a 50 cm3 beaker • a balloon
• a bottle of baking powder • a pair of safety goggles
(sodium carbonate)
• a bottle of vinegar (ethanoic acid)

© 2014 Marshall Cavendish Education Pte Ltd Activity 18 77

077-092 SS0281 (S)LSSPBB_18.indd 77 19/05/2014 14:21


Procedure and Observations
1. Put six to ten spatulas of baking powder into the glass bottle.

2. Wet the piece of cork with water.

3. Measure approximately 30–40 cm3 of vinegar. Transfer the vinegar into the bottle. Stopper
the bottle with the wet cork immediately.

Note:
Move away immediately from the stoppered bottle.

4. What do you obser ve?


Bubbles are produced when vinegar is added to baking powder. The cork pops out of the bottle.

Discussion
1. Describe the change that occurred when the two substances were mixed in the bottle.
A gas is produced when the two substances were mixed. This gas takes up space within the glass

bottle. As more gas is produced, the gas pressure builds up and pushes out the cork.

2. What would you obser ve if a deflated balloon was placed immediately over the mouth of the
glass bottle, instead of the piece of wet cork?
The gas produced from the reaction would fill up the balloon.

Extension
Based on your observations in the experiment, explain why it is unsafe to mix different types of
medication without consulting a doctor.
It is unsafe to mix different types of medication as chemical changes may occur when different types of

medication are mixed. The products of these chemical changes may cause harmful side effects to our body.

78 Chapter 18 © 2014 Marshall Cavendish Education Pte Ltd

077-092 SS0281 (S)LSSPBB_18.indd 78 20/5/14 8:30 AM


Name: ( )

Chapter 18
Suggested 2 periods
duration 70 minutes
Class: Date:

Activity
18.2 Changes Caused by Heat and Light

Chemical Changes
Aim
• To investigate the chemical changes caused by heat and light on different substances

Apparatus and Materials


Part A
• a bottle of copper carbonate • a test-tube holder
• two test tubes • a pair of safety goggles
• a test-tube rack • a Bunsen burner
• a spatula • a bottle of sugar

Part B
• two petri dishes • a piece of filter paper
• a bottle of sodium chloride solution • a white tile
• a bottle of silver nitrate solution • an opaque object (coin, paper clip)
• a pair of tongs • a table lamp (optional)

Part A: Changes Caused by Heat

Safety Precautions
The following are some important safety precautions:
• Wear safety goggles when heating substances.
• Move the test tube to and fro slightly over the flame during heating.
• Avoid pointing the test tube at anyone during heating.
• Wash the test tubes only after they have cooled down. This prevents them
from cracking.

Procedure and Observations

1. Obser ve the colour and state of the copper carbonate provided. Record your
obser vations in the table provided.

2. Place one spatula of copper carbonate into a clean, dr y test tube.

© 2014 Marshall Cavendish Education Pte Ltd Activity 18 79

077-092 SS0281 (S)LSSPBB_18.indd 79 19/05/2014 14:22


3. Heat the sample in the test tube using a non-luminous flame until there is no
further change.

4. Obser ve and record the changes (e.g. colour change, colour of gases, if
any) in the sample immediately after heating. Cool the sample and take
note of its colour.

5. Repeat steps 1 to 4 for the sugar sample.

6. Record your obser vations in the table below.

Process that
Appearance had taken
Appearance
Substance before Changes on heating place upon
on cooling
heating heating
(e.g. melting)
Copper Green powder Turns into a black solid Black powder Decomposition
carbonate

Sugar White crystals Melts into a yellow liquid, then Black solid Decomposition
turns into a black solid; sweet
smell given off; colourless droplets
of liquid on sides of the test tube

Discussion

Write a word equation for the chemical change that takes place when sugar is heated.
heat
Sugar Carbon + Water vapour

Part B: Changes Caused by Light

Safety Precautions
The following are some important safety precautions:
• Wear safety goggles.
• Avoid touching the silver nitrate solution with your fingers, as it leaves brown stains that are
difficult to remove.

1. Fill one petri dish with the sodium chloride sodium chloride silver nitrate
solution and the other with the silver nitrate solution solution
solution.

80 Chapter 18 © 2014 Marshall Cavendish Education Pte Ltd

077-092 SS0281 (S)LSSPBB_18.indd 80 19/05/2014 14:22


filter paper

2. Using a pair of tongs, fully immerse a filter sodium chloride silver nitrate
paper into the sodium chloride solution, solution solution
followed by the silver nitrate solution.

3. Place the filter paper on a clean white tile. white tile

4. Place an opaque object (e.g. a coin) on the


filter paper. Then place the white tile (with
the filter paper and opaque object) under
strong sunlight or under a table lamp for opaque object
about 15 minutes.

5. Trace the outline of the opaque object on the


filter paper. Remove the object from the dried
filter paper.

6. Paste the filter paper in the space provided,


and label the dark and light regions.

Paste and label your filter paper here.

Discussion

1. Based on the results you have found, deduce the hypothesis for the experiment in Part B.
Interactions between matter and light bring about chemical changes.

2. Dipping the filter paper in the sodium chloride solution followed by the silver nitrate solution
produces silver chloride. Explain how the reaction of silver chloride with light affects the filter
paper.
The silver chloride reacted in the presence of light to form a dark region. The area blocked by the

opaque object did not darken because silver chloride did not react in the absence of light.

© 2014 Marshall Cavendish Education Pte Ltd Activity 18 81

077-092 SS0281 (S)LSSPBB_18.indd 81 19/05/2014 14:22


Extension
1. What questions should we ask before we can conclude that a reaction is a chemical change?
Is the reaction permanent and irreversible? Did the substances react to form new substances? (Accept any

other reasonable answers.)

2. A control experiment is sometimes required to confirm the results of another experiment.

(a) Part B of the experiment was carried out to investigate the effect of light on treated
filter paper.

(i) State the variable that you changed for this investigation.
Light

(ii) When planning a control experiment, we can remove this variable to check if it is a
cause for the change. Using the procedure in Part B, list the steps taken to set up
a control experiment.
Immerse another filter paper of the same size into the sodium chloride solution, followed by

the silver nitrate solution. Place this filter paper on a white tile and place the tile in a dark

place. This is the control for the experiment.

(b) What is the expected result of this control experiment? How can we use this result to
confirm the result of the experiment?
The filter paper of the control experiment will remain unchanged. Compare the results from the control

experiment with those from the experiment on page 81. The results confirm that the presence of light

has caused the filter paper to darken.

82 Chapter 18 © 2014 Marshall Cavendish Education Pte Ltd

077-092 SS0281 (S)LSSPBB_18.indd 82 19/05/2014 14:22


Name: ( )

Chapter 18
Suggested 2 periods
duration 70 minutes
Class: Date:

Activity
18.3 Action of Acids on Metals and
Carbonates

Chemical Changes
Aims
• To investigate the effect of acids on common metals
• To investigate the effect of acids on carbonates
• To investigate the presence of carbonates in common substances around us

Apparatus and Materials


Part A
• a bottle of dilute hydrochloric acid • a bottle of magnesium strips
• a test tube • a test-tube stopper
• a test-tube rack • a wooden splint
• a test-tube holder • a bottle of zinc
• a thermometer • a bottle of iron filings

Part B
• a bottle of limewater • a bottle of dilute hydrochloric acid
• two test tubes • a delivery tube with stopper
• a test-tube rack • a test-tube holder
• a plastic spatula • a bottle of copper carbonate
• a bottle of calcium carbonate

Procedure and Observations

Part A: Investigating the Effect of Acids on Metals

1. Pour dilute hydrochloric acid into a clean test tube until it is about 2 cm deep. Measure the
temperature of the acid using a thermometer.

2. Place one magnesium strip into the acid. Record your obser vations.
Effer vescence/Tiny bubbles were produced. The strip of magnesium became smaller.

3. Use the thermometer to measure the temperature of the acid. Record temperature changes
as ‘increase’, ‘decrease’ or ‘remains constant’ in the table in Step 8.

4. Remove the thermometer and place your thumb over the mouth of the test tube. Shake the
test tube to mix the acid and metal well. Ensure that the mixture does not come into contact
with your thumb.

© 2014 Marshall Cavendish Education Pte Ltd Activity 18 83

077-092 SS0281 (S)LSSPBB_18.indd 83 19/05/2014 14:24


Testing for the presence of hydrogen gas

5. Light a wooden splint.

6. Remove the test-tube stopper from the test tube and insert the lighted/burning splint into
the mouth of the test tube.

What do you obser ve?


The lighted splint extinguishes with a ‘pop’ sound.

7. If the lighted splint extinguishes with a ‘pop’ sound, hydrogen gas is present.

8. Repeat steps 1 to 6 with other metals and record your obser vations in the table below.

Metal Temperature change Observation


Magnesium Increase Effervescence of colourless, odourless gas

Zinc Increase Effervescence of colourless, odourless gas

Iron Increase Effervescence of colourless, odourless gas

Discussion
What can you conclude about the reaction of acids with metals?
Acids react with metals to produce hydrogen gas.

Part B: Investigating the Effect of Acids on Carbonates

1. Pour limewater into a clean test tube until it is about 2 cm in depth.

2. Place three spatulas of calcium carbonate into another clean test tube.

3. Pour in dilute hydrochloric acid until it covers the solid (about 3 cm in depth). Shake the test
tube to mix the acid and carbonate well.

84 Chapter 18 © 2014 Marshall Cavendish Education Pte Ltd

077-092 SS0281 (S)LSSPBB_18.indd 84 19/05/2014 14:24


4. Stopper the mouth of the test tube (containing
calcium carbonate and dilute hydrochloric acid) with
the deliver y tube. Then dip the longer end of the
deliver y tube into the test tube of limewater. What
do you obser ve?

Effervescence was observed. Tiny bubbles were produced. The amount of calcium carbonate decreases.

The limewater turned milky.

5. When the limewater turns milky, carbon dioxide gas is present.

6. Pour away the contents of the test tube and wash the test tube. Repeat steps 1 to 6 with
copper carbonate and record your obser vations in the table below.
Colour of solution at
Carbonate Observation
end of experiment
Effervescence of colourless,
Calcium carbonate Colourless odourless gas
Effervescence of colourless,
Copper carbonate Blue odourless gas

Discussion
1. What can you conclude about the reaction of acids with carbonates?
Acids react with carbonates to produce carbon dioxide gas.

2. Suggest one possible indication that one metal is reacting faster than the other metal.
A metal that reacts faster would dissolve faster and cause bubbles to be produced at a faster rate.

3. State the products that are formed from the reaction of dilute hydrochloric acid with metals.
Salt and hydrogen gas

© 2014 Marshall Cavendish Education Pte Ltd Activity 18 85

077-092 SS0281 (S)LSSPBB_18.indd 85 19/05/2014 14:24


Extension
1. Some jewellery such as bangles may change colour when worn regularly. What are the possible
explanations for this?
Jeweller y may change colour due to various factors. These include exposure to warm and humid

environment. Sweat, which mixes with oil secreted by the skin to turn acidic, may also lead to colour

changes in jeweller y. Other reasons for colour changes in jeweller y include exposure to the chemicals

in per fumes, creams and soaps used ever y day.

2. Baking powder contains a form of carbonate. State the purpose of adding baking powder to
the cake mixture when baking a cake.
Baking powder produces bubbles of carbon dioxide gas that expand during baking. This helps the

cake to rise and become lighter.

3. Some buildings and stone statues are made of marble, which contains calcium carbonate.
What happens to these buildings and statues when acid rain comes into contact with them?
Calcium carbonate reacts with the acid rain and dissolves away. Hence, the buildings and stone statues

become corroded.

86 Chapter 18 © 2014 Marshall Cavendish Education Pte Ltd

077-092 SS0281 (S)LSSPBB_18.indd 86 19/05/2014 14:24


Name: ( )

Chapter 18
Suggested 2 periods
duration 70 minutes
Class: Date:

Activity
18.4 Properties of Acids

Chemical Changes
Aims
• To investigate the effect of acids on litmus paper
• To investigate the taste of organic acids commonly found in food

Apparatus and Materials


• a white tile • a bottle of dilute nitric acid
• a bottle of red and blue litmus paper • a bottle of lime/lemon juice
• a dropper • three strips of filter paper
• a bottle of dilute hydrochloric acid • a small lime (cut in half)/a slice of lemon
• a bottle of dilute sulfuric acid

Procedure and Observations


1. Rinse the white tile with tap water. Wipe the tile dr y.

2. Place one strip of red litmus paper and one strip of blue litmus paper side by side on the
white tile.

3. Use the dropper to place two drops of dilute hydrochloric acid onto each piece of litmus
paper.

4. Record the colour changes in the table provided on the next page.

5. Rinse the dropper and white tile thoroughly.

6. Repeat steps 1 to 5 for each acid provided.

dilute dilute dilute


hydrochloric sulfuric nitric
acid acid acid

7. Place one strip of red litmus paper and one strip of blue litmus paper side by side on the
white tile.

8. Squeeze a few drops of lime/lemon juice onto each piece of litmus paper.

9. Record the colour changes in the table provided on the next page.

© 2014 Marshall Cavendish Education Pte Ltd Activity 18 87

077-092 SS0281 (S)LSSPBB_18.indd 87 19/05/2014 14:24


Effect on blue Effect on red
Acid
litmus paper litmus paper
Dilute hydrochloric acid Turns red Remains red
Dilute sulfuric acid Turns red Remains red
Dilute nitric acid Turns red Remains red
Substance
Lime/Lemon juice Turns red Remains red

10. Squeeze a few drops of lime/lemon juice onto a clean piece of filter paper. Place the filter
paper at the tip of your tongue. Rinse your mouth. Then record the taste of the lime/lemon
juice in the space provided.

Taste of lime/lemon juice: Sour

Safety Precautions
The following are some important safety precautions:
• Never taste any chemicals in the laboratory unless instructed by your teacher.
• Inform your teacher if you have any chemical allergies.

Discussion
1. Based on your obser vations, explain why limes/lemons contain an acid.
Like acids, limes/lemons taste sour and turn blue litmus paper red.

2. Should we test for an acid by tasting it? Why? No. Some acids are corrosive and cause burns.

Extension

(a) Alpha hydroxyl acids (AHAs) are naturally occurring acids that can be found in sugar cane
juice and sour milk. These mild acids are used in chemical face peels and some skincare
products. Describe how AHAs work.
The mildly corrosive property of AHAs is used as an alternative to facial scrubs. This property

of AHAs also helps to remove dirt and kill bacteria on the skin.

(b) AHAs may not be suitable for some people. Why is this so? What are the possible negative
side effects of AHAs?
Acids are corrosive in nature. Although AHAs are mild acids, their corrosive property may be too

harsh for people with sensitive skin. This may lead to negative side effects such as thinning skin,

burns and rashes.

88 Chapter 18 © 2014 Marshall Cavendish Education Pte Ltd

077-092 SS0281 (S)LSSPBB_18.indd 88 19/05/2014 14:24


Name: ( )

Chapter 18
Suggested 2 periods
duration 70 minutes
Class: Date:

Activity
18.5 Properties of Alkalis

Chemical Changes
Aims
• To investigate the effect of alkalis on litmus paper
• To investigate how alkalis feel to the touch

Apparatus and Materials


• a white tile • a bottle of dilute calcium hydroxide
• a bottle of red and blue litmus paper • a bottle of aqueous ammonia
• a dropper • a beaker/test tubes
• a bottle of dilute sodium hydroxide • two strips of filter paper
• a bottle of dilute potassium hydroxide • a bottle of soap solution

Procedure and Observations


1. Rinse the white tile with tap water. Wipe the tile dry.

2. Place one strip of red litmus paper and one strip of blue litmus paper side by side on the white tile.

3. Use the dropper to place two drops of dilute sodium hydroxide onto each piece of litmus paper.

4. Record the colour changes in the table provided.

5. Rinse the dropper and white tile thoroughly.

6. Repeat steps 1 to 5 for each alkali provided.

dilute dilute dilute aqueous


sodium potassium calcium ammonia
hydroxide hydroxide hydroxide

© 2014 Marshall Cavendish Education Pte Ltd Activity 18 89

077-092 SS0281 (S)LSSPBB_18.indd 89 19/05/2014 14:26


Alkali Effect on red litmus paper Effect on blue litmus paper
Dilute sodium hydroxide Turns blue Remains blue
Dilute potassium hydroxide Turns blue Remains blue
Dilute calcium hydroxide Turns blue Remains blue

Aqueous ammonia Turns blue Remains blue

Safety Precautions
The following are some important safety precautions:
• Never taste any chemicals in the laboratory unless instructed by your teacher.
• Inform your teacher if you have any chemical allergies.

7. Wash a beaker and half fill it with tap water. Add two drops of diluted sodium hydroxide
into it. Dip a strip of filter paper into the solution. Rub the wet filter paper between your
fingertips. Describe how the alkali feels to the touch.

The dilute sodium hydroxide feels slippery/soapy .

8. Collect a little soap solution in a clean test tube. Tilt your test tube and dip the ends of the
strips of blue and red litmus paper into the soap solution. Record your obser vations below.

Effect on red litmus: Turns blue

Effect on blue litmus: Remains blue

9. What can you conclude about the properties of alkalis?


Alkalis turn red litmus paper blue.

Discussion
1. Can we test for an alkali by tasting it? Why?
No. Some alkalis are corrosive and cause burns.

2. What substance was present in the soap solution that was provided? Explain your answer.
An alkali. It turned red litmus paper blue and it feels soapy.

90 Chapter 18 © 2014 Marshall Cavendish Education Pte Ltd

077-092 SS0281 (S)LSSPBB_18.indd 90 19/05/2014 14:27


Extension
1. Alkalis react with acids in neutralisation reactions. Write down the equation that occurs
when hydrochloric acid reacts with sodium hydroxide. (Hint: Sodium chloride is produced.)
Hydrochloric acid + Sodium hydroxide → Sodium chloride + Water

2. (a) Toothpaste contains an alkali. How does brushing our teeth with toothpaste help to prevent
tooth decay?
When we consume food, bacteria ‘feeds’ on the food and produces acid on our teeth. This acid

will lead to tooth decay if left untreated. Brushing with an alkaline toothpaste helps to neutralise

the acid and prevent tooth decay.

(b) How can we test for alkali in toothpaste? Predict the result of the test.
We can test for alkali in toothpaste by mixing a little bit of toothpaste with water in a test tube

and dipping a red litmus paper in the toothpaste mixture. The litmus paper will turn blue.

3. (a) Lee was stung by a bee and he was told that a bee sting is acidic. Lee cleaned the
wound with calamine lotion. What is Lee’s hypothesis about calamine lotion?
Calamine lotion is alkaline.

(b) Assuming that Lee’s hypothesis is correct, how does calamine lotion work on the bee sting?
Calamine lotion neutralises the acid in the area that was stung. This helps to reduce the pain

and swelling from the acid present in the sting.

© 2014 Marshall Cavendish Education Pte Ltd Activity 18 91

077-092 SS0281 (S)LSSPBB_18.indd 91 19/05/2014 14:27


077-092 SS0281 (S)LSSPBB_18.indd 92 19/05/2014 14:27
Name: ( )

Chapter 19
Suggested
4 weeks
duration
Class: Date:

Activity
19.1 Effect of Light on Organisms

Interactions Within Ecosystems


Aims
• To obser ve the growth of duckweeds under different light intensities
• To record physical quantities (number) and qualities (colour of leaf) of duckweeds under different
light intensities over a four-week period
• To suggest suitable light intensities for the growth of duckweeds
• To predict other environmental factors that affect the growth of duckweeds

Apparatus and Materials


• 400 cm3 of culture solution • a table lamp (60 W)
• four 250 cm3 beakers • a light meter
• 80 duckweed plants

Note:
The apparatus and materials above are to be shared by four pupils.

Procedure and Observations


1. Fill each beaker with 100 cm3 culture solution.

2. Put 20 duckweed plants into each beaker.

3. Place the four beakers at different distances (e.g. 10 cm, 30 cm, 50 cm and 70 cm) from the
lamp. Switch on the lamp.

4. Use a light meter to measure the light intensity at each beaker. Record the light intensity in
the table below.

Distance of beaker from lamp (cm) Light intensity (cd)

(Answers vary.)

© 2014 Marshall Cavendish Education Pte Ltd Activity 19 93

093-108 SS0281 (S)LSSPBB_19.indd 93 19/05/2014 14:28


5. Count the number of duckweed leaves in each beaker. Record the number of leaves present
in the table below.

Distance of beaker from lamp (cm) Number of leaves present

(Answers vary.)

6. Examine the plants every week for four weeks. Record the number of leaves and any colour
change in the leaves.

Distance of Week 1 Week 2 Week 3 Week 4


beaker from
No. of Colour No. of Colour No. of Colour No. of Colour
lamp (cm)
leaves change leaves change leaves change leaves change

(Answers vary.)

Discussion
1. (a) Which light intensity is the most suitable for the growth of duckweeds?
The light intensity when the lamp is 10 cm away from the plant.

(b) Why is this light intensity most suitable for the growth of duckweeds?
Light is essential for photosynthesis. The rate of photosynthesis is higher when the light intensity

is higher.

94 Chapter 19 © 2014 Marshall Cavendish Education Pte Ltd

093-108 SS0281 (S)LSSPBB_19.indd 94 19/05/2014 14:28


2. (a) Which light intensity is the least suitable for the growth of duckweeds?
The light intensity when the lamp is 70 cm away from the plant.

(b) Why is this light intensity least suitable for the growth of duckweeds?
The rate of photosynthesis is lower when the light intensity is lower.

3. Name two other environmental factors that affect the rate of growth of duckweeds.
Temperature and the pH of the solution

Extension
1. Mar y conducted the same experiment but changed the distances to 50 cm, 75 cm, 100 cm
and 125 cm. She obser ved the plants ever y day for five days. After five days, she still did
not obser ve any notable changes on the plants. What do you think was Mar y’s mistake in
the experiment?
Mary should have observed the plants for a longer period of time. Five days might not be enough for the

plants to produce new leaves. Thus, she was not able to observe any changes.

2. Give two examples of plant adaptations in response to light.


Some plants have large leaves to capture more sunlight for photosynthesis. In forests where trees grow

close together, sunlight does not reach the trees near the ground. Some trees have adapted to grow

very tall in order to reach the sunlight above the forest. (Accept any other reasonable answers.)

3. Light affects animals as well as plants.

(a) Katie came up with the hypothesis that some animals are sensitive to light and thus avoid
light when they can. List two questions that Katie needs to ask before she carries out an
experiment to test her hypothesis.
What organism should I use? How would I control the light that the organism receives?

(Accept any other reasonable answers.)

© 2014 Marshall Cavendish Education Pte Ltd Activity 19 95

093-108 SS0281 (S)LSSPBB_19.indd 95 19/05/2014 14:28


(b) Katie obtained four earthworms, a lamp and a shallow, elongated cardboard box.
Write down the steps for Katie’s experiment.
1. Position the lamp so that it shines only onto the right side of the box.

2. Place one earthworm in the middle of the box.

3. Observe the direction of the earthworm’s movements.

4. Using a different earthworm, repeat steps 1 to 3 but position the lamp so that it shines only

onto the left side of the box.

5. Using a different earthworm, repeat steps 1 to 3 but position the lamp so that it shines onto

the middle part of the box.

6. Using a different earthworm, repeat steps 1 to 3 but turn off the lamp.

96 Chapter 19 © 2014 Marshall Cavendish Education Pte Ltd

093-108 SS0281 (S)LSSPBB_19.indd 96 19/05/2014 14:28


Name: ( )

Chapter 19
Suggested
4 weeks
duration
Class: Date:

Activity
19.2 Effect of Acidity and Alkalinity on Organisms

Interactions Within Ecosystems


Aim
• To investigate the effect of pH on the growth of duckweeds

Apparatus and Materials


• buffer tablets (one of pH 2, 4, 6 and 8 per group)
• four 250 cm3 beakers
• 400 cm3 of culture solution
• 80 duckweed plants

Note:
The apparatus and materials above are to be shared by four pupils.

Procedure and Observations


1. Dissolve one buffer tablet (pH 2) in a beaker containing 100 cm3 of culture solution.

2. Place 20 duckweed plants in the culture solution.


duckweed plants

culture solution

3. Repeat steps 1 and 2 using buffer tablets of pH 4, 6 and 8.

4. Place the four beakers of duckweeds near the window where they can receive sunlight.

Note:
Do not place the duckweeds under direct sunlight.

© 2014 Marshall Cavendish Education Pte Ltd Activity 19 97

093-108 SS0281 (S)LSSPBB_19.indd 97 19/05/2014 14:28


5. Count the number of duckweed leaves present in each beaker.
Record this in the table below.

pH Number of leaves present

8
(Answers vary.)

6. Examine and count the number of duckweed leaves ever y week for four weeks.
Record your obser vations in the table below.

Week Number of leaves present


pH 2 pH 4 pH 6 pH 8

(Answers vary.)

Discussion
1. Which pH solution is the most suitable for the growth of duckweeds?
The pH solution which is near neutral

2. Which pH solution is the least suitable for the growth of duckweeds?


The solution with pH 2

3. Why is culture solution used instead of distilled water?


It provides mineral salts essential for plant growth.

98 Chapter 19 © 2014 Marshall Cavendish Education Pte Ltd

093-108 SS0281 (S)LSSPBB_19.indd 98 19/05/2014 14:28


4. Predict the results if you used distilled water instead of the culture solution in the experiment.
Explain your prediction.

The mineral salts in the culture solution would help the duckweed plants to grow faster. Distilled water

does not contain mineral salts. Thus, the duckweed plants would not grow as well if they were placed

in distilled water.

5. Would the results be the same if you used a different plant species? Explain your answer.
No. Different plant species have different tolerance to acidity and alkalinity.

Extension
1. How can you improve the experiment to obtain more reliable results?
Have at least two set-ups with the same number of duckweeds for each pH. At the end of the experiment,

take the average reading (number of leaves present) of both set-ups for each pH.

2. (a) Henry noticed that a certain shrub is always present in the area where the carnivorous
plant Venus flytrap grows. He knows that the Venus flytrap grows well in acidic soil.
However, he noticed that this shrub also grows in areas where the Venus flytrap does not
grow. What hypothesis can Henr y formulate from his obser vations?
The shrub can tolerate a wider range of pH than the Venus flytrap.

(b) How can Henr y test his hypothesis?


He can take soil samples from the areas where both the shrub and the Venus flytrap grow. He

can also take soil samples from areas where the shrub grows but the Venus flytrap does not.

He can then find out the pH of the soil samples. By comparing the pH levels of the different soil

samples, Henr y can test his hypothesis.

3. Before planting a vegetable garden, what are the questions to consider regarding pH levels?
What is the pH level of the soil? What is the most suitable pH level for this kind of vegetable?

What can I do to adjust the pH level of soil? (Accept any other reasonable answers.)

© 2014 Marshall Cavendish Education Pte Ltd Activity 19 99

093-108 SS0281 (S)LSSPBB_19.indd 99 19/05/2014 14:29


4. Jamie monitored the pH level of water in a small pond. His readings showed that the water
was more alkaline during daytime than during nighttime. Suggest a possible reason for this.
There could be many aquatic plants present in the small pond. Plants use the dissolved carbon dioxide in

water to carry out photosynthesis. This makes the water less acidic. Since photosynthesis occurs only in

the presence of sunlight, the water in the pond is more alkaline during daytime than during nighttime.

5. Some farmers add calcium hydroxide (slaked lime) to soil to make it more alkaline. What do
you think might be an advantage of changing the alkalinity of the soil?
Some plants such as cabbage grow well in slightly alkaline soil. Hence, making the soil more alkaline

would help the plant grow more. This would help farmers grow more crops.

100 Chapter 19 © 2014 Marshall Cavendish Education Pte Ltd

093-108 SS0281 (S)LSSPBB_19.indd 100 19/05/2014 14:29


Name: ( )

Chapter 19
Suggested 1 period
duration 35 minutes
Class: Date:

Activity
19.3 Simulation of the Transfer of Energy
in a Food Chain

Interactions Within Ecosystems


Aim
• To simulate the interactions of organisms in a food chain

Apparatus and Materials


• two pieces of blank recycled paper per student
• one marker pen/crayon
• a pair of scissors
• one masking tape
• one sandwich-size plastic bag per student
• one bag of peanuts with shells
• seven plastic containers of different colours (two red, two blue and three white)

Procedure and Observations


Background Information:
This is a simulation to show the feeding relationships within a food chain.
Students will represent mice, snakes or hawks.

Mouse Snake Hawk

Students must follow the following rules strictly to achieve the objective of this activity.
• No fighting/pushing/kicking/elbowing.
• For a mouse to sur vive, it must fill its ‘stomach’ to at least half full.
• For a snake to survive, it must fill its ‘stomach’ to the top (leave enough space to close the bag).
• For a hawk to sur vive, it must collect at least two full bags of peanuts.
• Hawks do not eat mice.
• Only mice are able to take the peanuts directly as food.

© 2014 Marshall Cavendish Education Pte Ltd Activity 19 101

093-108 SS0281 (S)LSSPBB_19.indd 101 19/05/2014 14:29


1. Assign each student to play the role of an animal.

2. Write down the role of the animal which each student is representing on the blank papers
provided. Using the masking tape, attach the papers onto the back and front of the student’s
shirt.

3. All students will receive one plastic bag except those who are representing hawks. Students
who are representing hawks will receive two plastic bags. This plastic bag will represent the
stomach of an animal. During the game, students will put their ‘food’ (peanuts with shells) into
their ‘stomachs’ (plastic bags).

4. The teacher will assign an area where the game will take place. The game must only take place
within this area.

5. The game will only begin when the teacher rings the bell. The first ring of the bell indicates the
start of the game. The second ring indicates the end of the game.

6. Place the peanuts in three white containers scattered around the game area. When the
game begins, the mice must collect as many peanuts as they can from the containers into
their stomachs. They must also tr y to avoid being eaten by the snakes.

7. The snakes will try to capture the mice.

8. When a snake captures a mouse, the mouse will empty the peanuts into the snake’s ‘stomach’.

9. At the same time, the hawks will try to capture snakes.

10. When a hawk captures a snake, he/she will collect all the contents in the snake’s ‘stomach’.

11. When an ‘animal’ captures another ‘animal’, he/she must wait ten seconds before capturing
another ‘animal’.

12. Place four empty containers — two blue and two red ones — in different spots in the game area.
An ‘animal’ must place a handful of peanuts each on one red and one blue container every time
he/she captures another ‘animal’.

13. ‘Animals’ that have been captured will leave the game area and sit on the side to watch the game.

14. The whole game should last for five minutes or until all the mice have been eaten, whichever
happens first. All ‘animals’ must try their best to survive during the game.

102 Chapter 19 © 2014 Marshall Cavendish Education Pte Ltd

093-108 SS0281 (S)LSSPBB_19.indd 102 19/05/2014 14:29


Note for teachers:
After one round of the game, ask the students to suggest other rules that they would like to add.

E.g.
– The snake can only eat the mouse when the mouse has half a bag full of peanuts.
– The hawk can only eat the snake when the snake has a bag of peanuts.
– There must be a shelter area for the mice to hide from the predator.
– The mice can only hide in the shelter area for no more than 30 seconds.
– Only a fixed number of mice can hide in the shelter.

Discussion
1. What do the peanuts in the simulation represent?
The source of energy of the organisms

2. Identify which of the animals in the simulation is a herbivore, carnivore or omnivore.


Mice are omnivores. Snakes and hawks are carnivores.

3. Identify which of the animals in the simulation is a predator or a prey.


Mice are preys of snakes. Snakes are predators of mice and preys of hawks. Hawks are predators of

snakes. No animal in the simulation preys on hawks.

4. Each ‘animal’ places peanuts on the blue and red containers every time he or she captures
another ‘animal’. What does this represent in a real food chain?
This represents that some energy is lost to the environment. Some energy is lost as heat during

respiration and as excretory products. Hence, the amount of energy decreases along the food chain.

5. Based on your observations in the simulation, what can cause an imbalance in the food chain?
An imbalance in the food chain is caused when a whole population of species is wiped out. In the

simulation, the food chain will stop when there is no more mouse left.

© 2014 Marshall Cavendish Education Pte Ltd Activity 19 103

093-108 SS0281 (S)LSSPBB_19.indd 103 19/05/2014 14:29


Extension
1. Predict what would happen if a snake-eating eagle was added to the food chain.
The snake population will decrease as more predators hunt them. The mouse population would then

increase as there are less snakes to hunt them.

2. If an increase in food supply caused the mouse population to grow, what effect would this
have on the snake and hawk populations?
An increase in mouse population will mean more food source for snakes. Thus, the snake population will

also increase. This will then lead to an increase in hawk population as the hawks feed on snakes.

3. How would the mouse population be affected if all the hawks were captured by humans?
The snake population would increase since there were no hawks to prey on them. This would lead to a

decrease in mouse population since there would be more snakes to prey on the mice.

4. In the simulation, the hawks have no predators. When hawks die, how are the nutrients inside
them recycled back into the environment?
Decomposers in the soil break down the nutrients in the dead bodies of the hawks. Through this

process, carbon dioxide is released into the atmosphere and mineral salts are released into the soil.

104 Chapter 19 © 2014 Marshall Cavendish Education Pte Ltd

093-108 SS0281 (S)LSSPBB_19.indd 104 19/05/2014 14:29


Name: ( )

Chapter 19
Suggested 2 periods
duration 70 minutes
Class: Date:

Activity
19.4 Photosynthesis and Respiration

Interactions Within Ecosystems


Aim
• To compare the gases given off during photosynthesis and respiration

Fresh Morning Air

Most people generally prefer to exercise outdoors during the day


than during the night. One of the reasons could be the higher
concentration of oxygen present in the air during the day than
during the night.

In this activity, we will investigate whether this is true by comparing


the gases given off by plants in the presence and absence of light.

Apparatus and Materials


• water • two spatulas of sodium hydrogen carbonate
• two test tubes per group
• two beakers • a wooden splint
• two filter funnels • a Bunsen burner
• two pieces of freshwater plants • a spatula
(e.g. hydrilla)

Procedure and Observations


1. Set up two experiments as shown below. Name the set-ups A and B.

test tube

water saturated with


beaker
carbon dioxide

filter funnel
water plant

wooden support
Set-up A Set-up B
(to be kept in sunlight) (to be kept in the dark)

© 2014 Marshall Cavendish Education Pte Ltd Activity 19 105

093-108 SS0281 (S)LSSPBB_19.indd 105 19/05/2014 14:30


2. In each beaker, dissolve one spatula of sodium hydrogen carbonate in the water. This
provides the plant with carbon dioxide.

3. Place Set-up A in the sunlight and Set-up B in the dark. You will notice that gas bubbles form
on the leaves of the plant that was placed in sunlight. These bubbles will rise up the test
tube and displace the water downwards.

4. When the test tube in Set-up A is about half-filled with gas, remove the test tube by placing
a thumb over its mouth.

5. Test the gas with a glowing splint. What happens?


The splint is rekindled or burst into flames.

If the splint rekindles, oxygen gas is present.

6. What do you observe in Set-up B?


No gas was collected in the test tube in Set-up B.

Discussion
1. Why was there gas collected in the test tube in Set-up A but none in the test tube in Set-up B?
The gas collected in Set-up A is oxygen. Oxygen is a product of photosynthesis. Set-up A was placed

under the sunlight. Thus, the plant was able to carry out photosynthesis. Since Set-up B was placed in

the dark, photosynthesis could not occur. Thus, no oxygen is released.

2. How would the results of the experiment be affected if you used a hydrilla plant with only a
few leaves?
Only a small amount of oxygen will be collected in the test tube in Set-up A. This is because photosynthesis

occurs in the leaves of plants.

3. What can you conclude from this experiment?


Based on the results of the experiment, it can be concluded that oxygen is given off during photosynthesis

but not during respiration.

106 Chapter 19 © 2014 Marshall Cavendish Education Pte Ltd

Blue litmus paper remains blue.


093-108 SS0281 (S)LSSPBB_19.indd 106 19/05/2014 14:30
4. The elodea plant is usually found in aquariums. If you had used elodea plants instead of hydrilla
plants in this experiment, would you have the same conclusion? Explain your answer.
Yes. The products of respiration and photosynthesis are always the same regardless of the organism that

carries out these processes.

Extension
1. Two students made differing statements:

Student A: Photosynthesis occurs only during daytime and respiration occurs only during
nighttime.
Student B: Photosynthesis occurs only during daytime. However, respiration occurs all
the time.

Which student made the correct statement? Explain your answer.


Student B. Photosynthesis occurs only during daytime. This is because light energy is needed for

photosynthesis to occur. On the other hand, respiration occurs all the time. Only food molecules and

oxygen are needed for respiration to occur. Light energy is not a condition needed for respiration.

2. Naomi prepared two set-ups similar to Set-up A in this experiment. Naomi added sodium
hydrogen carbonate into one of her set-ups and labelled it Set-up X. When dissolved, sodium
hydrogen carbonate releases carbon dioxide into the water. The other set-up, Set-up Y,
contained only distilled water.

(a) What was Naomi tr ying to find out?


The effect of carbon dioxide concentration on photosynthesis

(b) Both set-ups were placed in the sunlight. Predict the results of Naomi’s experiment.
Explain your prediction.
More oxygen will be collected in the test tube in Set-up X than in Set-up Y. Sodium hydrogen

carbonate provides carbon dioxide to the water in Set-up X. The plant in Set-up X now has more

carbon dioxide to use for photosynthesis. Thus, more oxygen is released into the test tube.

© 2014 Marshall Cavendish Education Pte Ltd Activity 19 107

093-108 SS0281 (S)LSSPBB_19.indd 107 19/05/2014 14:30


3. Air is composed of a mixture of gases — 78% nitrogen, 21% oxygen and traces of water
vapour, carbon dioxide and other gases. Most organisms, including humans, need oxygen to
sur vive.

(a) How are the gases involved in respiration exchanged between an organism and its
surroundings?
During breathing, the oxygen needed for respiration is taken in from the surroundings and into the

organism. Carbon dioxide, a product of respiration, is released by the organism into the surroundings.

(b) How do flowering plants conduct the process mentioned in (a)?


Plants exchange gases with the surroundings through their stomata.

4. Rui Yi used a data logger to monitor the dissolved oxygen concentration in a lake. Fish and
other aquatic organisms in the lake breathe in the dissolved oxygen to stay alive.

(a) Rui Yi’s data showed that the dissolved oxygen concentration increases during daytime.
Suggest a possible reason for this.
During daytime, the aquatic plants in the lake conduct photosynthesis. Plants release oxygen during

photosynthesis. Hence, the dissolved oxygen concentration in the lake increases.

(b) Rui Yi’s data also showed that the dissolved oxygen concentration decreases during
nighttime. Suggest a possible reason for this.
During nighttime, aquatic plants and animals continue to respire. During respiration, these organisms

take in the dissolved oxygen in the water. Also, the plants do not release oxygen into the water, as

photosynthesis does not occur during nighttime. Thus, the dissolved oxygen concentration in the

lake decreases.

108 Chapter 19 © 2014 Marshall Cavendish Education Pte Ltd

093-108 SS0281 (S)LSSPBB_19.indd 108 19/05/2014 14:30


Lower Secondary

Lower Secondary
Express/Normal( A )
2nd Edition

The Science Matters for Express/Normal( A ) Practical Book


( 2nd Edition ) is designed to complement the Science Matters for
Express/Normal(A) (2nd Edition) textbook. It is written in line with the 2013
Lower Secondary Science syllabus from the Ministry of Education, Singapore. 2nd Edition

Practical Book Teacher's Edition


The practical book enhances the understanding of concepts that students learnt
and provides opportunities for the development of skills.

In the practical book, hands-on activities are used to encourage students to


explore scientific concepts through experimentation. These hands-on activities
develop process skills that students need in order to carry out scientific inquiry.
Selected experiments begin with real-life contexts so that students can develop
an understanding of how the abstract scientific concepts learnt are connected to
real-life occurrences.

Develops process skills in students


The skills developed in each activity are clearly outlined so that students are
made aware of the process of scientific inquiry that occurs in the activity.

Volume B
This leads to a greater understanding of scientific knowledge and promotes the
spirit of inquiry in students.

Promotes Science as an inquiry

2nd Edition
The activities encourage students to discover concepts on their own through
experimentation. These activities have been carefully designed to develop
students to think in a systematic and logical manner about the world
around them.

J Fong • Lam P K • E Lam • C Lee • Loo P L


Other components
• Textbook
• Workbook

Joan Fong • Lam Peng Kwan • Eric Lam


ISBN 978-981-01-1747-4 Christine Lee • Loo Poh Lim

Practical Book
Volume B Teacher's Edition
(S)LSSBPBTE_Cover.indd 1 9/12/12 5:30 PM

Potrebbero piacerti anche